Download as pdf or txt
Download as pdf or txt
You are on page 1of 99

TOEFL PREPARATION EXERCISES

For University Student

Compiled from
Baron’s Practice Exercises For The TOEFL
Fifth Edition By Pamela J. Sharpe, Ph.D
&
Longman Preparation Course For The TOEFL
By Deborah Phillips

UIN SATU TULUNGAGUNG 2022

i
TOEFL Preparation Exercises Structure and Written Expression

EXERCISE 1: Sentences—Verbs
In some sentences in the Structure Section on the Paper -Based TOEFL or the
Computer-Based TOEFL, you will be asked to identify the correct verb. In fact,
most of the sentences in the Structure Section are verb problems. A verb is a word or
phrase that expresses action or condition. A verb can classified as transitive or
intransitive according to whether it requires a complement; it can be classified
further according to the kind of complement it requires, including not only noun s,
pronouns, adjective and adverbs, but also -ing forms or infinitives. Choose the
correct answer in the incomplete sentence. Choose the incorrect word or phrase in
the underlined choices.
1. Almost everyone fails ……. the driver's test on the first try.
(A) passing
(B) to have passed
(C) to pass
(D) in passing

2. When the silkworm gets through to lay its eggs, it dies.


(A) (B) (C) (D)

3. If endangered species _____ saved, rain forests must be protected.


(A) are to be
(B) be
(C) can be
(D) will be
4. The average spoken sentence in conversational English takes 2.5 seconds _______
(D) for to complete
(B) completing
(C) to complete
(D) by completing

5. Only twenty years ago, most doctors agreed _____truthful with their terminally ill
patients, a trend that has reversed itself in modem medical practice.
(A) don't to be
(B) not to be
(C) we shouldn't been
(D) not to been

6. William Torrey Harris was one of the fi educators interested _____ a logical
progression of topics in the school curriculum.
(A) in establishing
(B) for establishing
(C) establishing
(D) to establish

48
TOEFL Preparation Exercises Structure and Written Expression
7. North American Indian tribes used sign language with tribes that spoke a different
language or dialect.
(A) to communicating
(B) for communicate
(C) to communicate
(D) for communicated
8 . Art tends to be………more after the death of the artist, but most literary works tens
decrease in value when the writer dies.
(A) price
(B) worthy
(C) worth
(D) value
9. Adult eagles let their offspring nests near their original nesting area.
(A) build
(B) builds
(C) building
(D) to build

10. A barometer is a device with a sealed metal chamber designed to reading the
(A) (B) (C)
change in the pressure of air in the atmosphere.
(D)

11. If a person does not have an attorney, the court _________ one.
(A) will appoint
(B) appointed
(C) would appoint
(D) appointing
12. Since lightning was probably significant in the formation of life, understanding it
(A)
might help us to understanding life itself.
(B) (C) (D)

13. Iowa ___ of flat-topped hills erected by the ancient Mound Builder people as
temples and burial sites.
(A) with a larger number
(B) has a large number
(C) having a large number
(D) a large number
14. If the oxygen supply in the atmosphere was not replenished by plants, it would
(A) (B) (C)
soon be exhausted.
(D)

49
TOEFL Preparation Exercises Structure and Written Expression
15. ___ the eight Ivy League schools are among the most prest igious colleges in the
United States.
(A) It is generally accepted that
(B) That it is accepted
(C) Accepting that it is
(D) That is accepted
16. The Girl Scouts, found by Juliette Gordon Low in 1912, has grown to a current
(A) (B) (C)
membership of more than three million girls.
(D)
17. To relieve pain caused by severe burns, prevent infection, and treat for shock,
______immediate steps.
(A) taking
(B) to take
(C) taken
(D) take
18. If gasoline vapor ………. with air, combustion will occur.
(A) mixed
(B) had mixed
(C) mixes
(D) mixture
19. Vermont, commonly known as the Green Mountain State, refused ______until
1791.
(A) to join the Union
(B) joining the Union
(C) the joining of the Union
(D) join the Union
20. Air constricted between the vocal chords makes them _____ producing sounds.
(A) to vibrate
(B) vibrating
(C) vibrate
(D) the vibration

50
TOEFL Preparation Exercises Structure and Written Expression

EXERCISE 2 : Sentences—Auxiliary Verbs

In some sentences in the Structure Section on the Paper-Based TOEFL or


Computer-Based TOEFL, you will be asked to select the correct auxiliary verb. An
auxiliary verb is a verb that accompanies the main verb and makes distinctions
in the meaning of the main verb. Some examples of auxiliary verbs are BE,
HAVE, or a modal. Choose the correct answer in the incomplete sentences. Choose
the incorrect word or phrase in the underlined choices.
1. On the average, a healthy heart _____ to pump five tablespoons of blood with
every beat.
(A) must
(B) ought
(C) can
(D) should
2. It is generally believed that Thomas Jefferson was the one who had researched and
(A)
wrote the Declaration of Independence during the months prior to its signing
(B) (C)
in July 1776.
(D)

3. In general, by the second year of production, the price of a new piece of technology
_______ significantly.
(A) will decreased
(B) has decreased
(C) will have decreased
(D) will has decreased
4. Although some higher structures have been build in New York City, none
(A) (B)
characterizes the skyline better than the Empire State Building.
(C) (D)
5. Research in genetics and DNA having had a profound influence on the direction of
(A) (B) (C)
treatment for a large number of diseases.
(D)
6. Bones composed chiefly of calcium, phosphorous, and a fibrous substance known as
(A) (B) (C) (D)
collagen.

51
TOEFL Preparation Exercises Structure and Written Expression
7. A cure for juvenile diabetes _____ until more funds are allocated to basic research.
(A) won't develop
(B) aren't developing
(C) don't develop
(D) won't be developed
8. During the past decade, twenty million college graduates spended more then fifty
(A) (B) (C)
billion dollars in ten-year student loans.
(D)

9. Civil engineers had better ___ steel supports in concrete structures built on unstable
geophysical sites.
(A) include
(B) including
(C) inclusion
(D) included
10. There is no limit to the diversity to be finding in the cultures of people throughout
(A) (B) (C) (D)
the world.

11. The cones of pine trees ______ two or three years to reach maturity.
(A) to take
(B) taking
(C) may take
(D) takes
12. The government requires that a census taken every ten years so that accurate statistics
(A) (B) (C)
may be compiled.
(D)

13. It is important that cancer is diagnosed and treated as early as possible in order
(A) (B) (C)
to assure a successful cure.
(D)

14. Although the scientific community had hoped that the field of transplantation -
____, the shortage of organ donors has curtailed research.
(A) progress
(B) had progressed

52
TOEFL Preparation Exercises Structure and Written Expression
(C) would progress
(D) progressing

15. Before railroad lines were extended from Missouri to New Mexico, millions of
(A) (B)
dollars in trade was used to be carried over the Santa Fe Trail by wheeled wagons.
(C) (D)
16. Based on a decline in vehicular deaths firing the past decade, seat belts, air bags,
baby seats, and other safety features in newer automobiles must be save lives.
(A) (B) (C) (D)
17. The gold used in jewelry is not strong enough unless it be alloyed.
(A) (B) (C) (D)

18. Even without strong wings, the ostrich has survived because it ____ at high speed
to escape predators.
(A) to run
(B) can run
(C) running
(D) run
19. General damage that been caused by aphids or pollution is sometimes known as
(A) (B) (C) (D)
blight.

20. Fred Astaire is said to had been the most popular dancer of his time, but was also
(A) (B) (C)
a talented actor, singer, and choreographer.
(D)

53
TOEFL Preparation Exercises Structure and Written Expression
EXERCISE 3: Sentences—Nouns

In some sentences in the Structure Section on the Paper -Based TOEFL or the
Computer-Based, TOEFL, you will be asked to identify the correct noun. A noun is
a word that names persons, objects, and ideas. There are two basic classifications of
nouns in English: count nouns and noncount nouns.
Count nouns are those that can be made plural by - s , - e s , or an irregular form.
They are used in agree ment with either singular or plural verbs. Noncount nouns
are those that cannot be made plural in these ways. They are used in agreement
with singular verbs. It is necessary to know whether a noun is count or noncount
to maintain verb agreement and to choose correct adjective modifiers. Choose the
correct answer in the incomplete sentences. Choose the incorrect word or phrase in the
underlined choices.

1. The understanding electricity depends on a knowledge of atoms and the subatomic


(A) (B) (C)
particles of which they are composed.
(D)
2. The U.S. Postal Service delivers more mails in one day than Federal Express does in
(A) (B) (C) (D)
one year.
3. The two main ____ are permanent magnets and electromagnets.
(A) kinds of magnets
(B) kind of magnets
(C) kind magnets
(D) kinds magnets
4. When water is frozen, it becomes _________
(A) ice
(B) ices
(C) the ice
(D) an ice
5. ______ can live to be more than fifteen years old.
(A) That it is dogs
(B) That dogs
(C) Dogs that
(D) Dogs
6. One of the most distinctive plant found in the desert is the Saguaro cactus.
(A) (B) (C) (D)

54
TOEFL Preparation Exercises Structure and Written Expression
7. In the fall, most trees lose ______, which have, by then, turned from green to gold
and orange.
(A) their leaf
(B) their leaves
(C) the leaf
(D) the leafs

8. Doctors have concluded that in addition to regular exercise, a diet rich in __ is


good for the heart.
(A) fruits and vegetable
(B) a fruit and vegetable
(C) the fruits and vegetables
(D) fruit and vegetables
9. A thunder usually follows lightning by five seconds for every mile between the flash
(A) (B) (C) (D)
and the observer.
10. Canada stretches from the Atlantic Ocean to the Pacific Ocean, and covers ___ of
almost four million square miles.
(A) a area
(B) an area
(C) the area
(D) area
11. During the early nineteenth century, ____ were hunted for their pelts.
(A) beaver
(B) beavers
(C) the beaver
(D) that beavers

12. The stories of Dr. Seuss have been enjoyed by millions of childrens.
(A) (B) (C) (D)
13. Collections of the simple and functional Shaker furniture can be seen in museum
(A) (B) (C) (D)
throughout the United States.

14. The decathlon is a two-day athletic competition which consists of ten types track
(A) (B) (C) (D)
and field events.
15. __ designs on a wall, also called graffiti, has become associated with gang activity
in many neighborhoods.
(A) Spraying of

55
TOEFL Preparation Exercises Structure and Written Expression
(B)The spraying of
(C) Spray the
(D) Sprays
16. __ by the author John Grisham are frequently on the best seller list.
(A) The novel
(B) Novels
(C) A novel
(D) Some novel
17. ___ have made communication faster and easier through the use of e-mail
and the Internet is widely recognized.
(A) It is that computers
(B) That it is computers
(C) Computers that
(D) That computers
18. Provide pensions for retired persons is the primary function of the social security
(A) (B) (C) (D)
system.

19. New equipments for medical diagnosis have made many formerly unpleasant
(A) (B) (C)
procedures quite painless.
(D)
20. Termites can do _ to the wood in homes before they are detected.
(A) an extensive damage
(B) extensive damages
(C) the extensive damage
(D) extensive damage

56
TOEFL Preparation Exercises Structure and Written Expression

EXERCISE 4: Sentences—Pronouns

In some sentences in the Structure Section on the Paper-Based TOEFL or the


Computer-Based TOEFL, you will be asked to identify the correct pronoun. A
pronoun is a word that can be used instead of a noun, usually to avoid repeating the
noun. A pronoun may be singular or plural; masculine, feminine, or neuter; and
first, second, or third person to agree with the noun to which it refers. A pronoun
may be used as the subject of a sentence or a clause or as the object of a sentence, a
clause, or a preposition. In English, pronouns are also used to express possessives
and reflexives. Choose the correct answer in the incomplete sentences. Choose the
incorrect word or phrase in the underlined choices.

1. The crime rate has begun to decline in New York City due to efforts on the
part of both government and private citizens to curb
(A) them
(B) him
(C) its
(D) it
2. Sloths spend most of its time hanging upside down from trees and feeding on leaves
(A) (B) (C)
and fruit.
(D)

3. When the European settlers came in the seventeenth century, the newcomers began
(A)
a systematic effort to push the Native Americans into the wilderness and to take
(B)
their land from their.
(C) (D)

4. Seals can ___ because they have a thick layer of blubber under their fur.
(A) keep them warm
(B) keep themselves warm
(C) they keep warm
(D) keep their warm
5. After Dr. Werner Arber discovered restriction enzymes, Drs. Daniel Nathan,
(A)
Hamilton Smith, and him were awarded the Nobel Prize for their research in that
(B) (C) (D)
field.

57
TOEFL Preparation Exercises Structure and Written Expression
6. There are not many people which adapt to a new culture without feeling some
(A) (B) (C) (D)
disorientation at first.

7. In order for people who speak different languages to engage in trade ______, they
often develop a simplified language caller pidgin.
(A) with each the other
(B) with each to the other
(C) with each another
(D) with each other
8. Those of us who have a family history of heart disease should make yearly
(A) (B) (C)
appointments with their doctors.
(D)
9. Domestic cats often show loyalty to their owners by leaving freshly killed prey suet
as birds for _______to find.
(A) they
(B) he
(C) them
(D) their
10. The United States and Canada have many trade agreements that benefit _______.
(A) one the other
(B) other
(C) other one
(D) each other
11. George Herman Ruth, which was better known as Babe Ruth, began his baseball
(A) (B) (C) (D)
career in 1914 with the Baltimore Orioles.

12. The constellation Orion is easily recognized by ______ three vertical stars.
(A) your
(B) its
(C) their
(D) her
13. The first full-length animated movie, Snow White, was produced by Walt Disney
(A) (B)
whom creative genius also inspired such animated classics as Bambi and Cinderella.
(C) (D)

58
TOEFL Preparation Exercises Structure and Written Expression
14. Wolves, which are known to travel in packs, both provide for and defend ______
through group cooperation.
(A) himself
(B) themselves
(C) itself
(D) theirselves
15. Although orchids give the appearance of being very fragile, they are actually very
hardy plants ______ indoors during the winter months.
(A) which may be grown
(B) what may grow
(C) who may be grow
(D) where may be growing
16. Hyperactivity in children may result from ______ some food additives.
(A) their eating
(B) they eat
(C) to eat
(D) them eating
17. In advanced stages of anorexia, the patient is unable to feed themselves.
(A) (B) (C) (D)
18. It is documented that Custer led his troops into a ravine near the Little Big Horn,
(A) (B)
where White, where a huge army of Sioux Indians was waiting for they.
(C) (D)
19. The sea horse is unique among fish because the female deposits their eggs in a
(A) (B) (C)
pouch that the male carries until the small sea horses are hatched.
(D)
20. Hawkeye was a character _______ James Fenimore Cooper created for The Last
of the Mohicans
(A) who
(B) whom
(C) which
(D) whose

59
TOEFL Preparation Exercises Structure and Written Expression

EXERCISE 5: Sentences—Modifiers
In some sentences in the Structure Section on the Paper-Based TOEFL or the
Computer-Based TOEFL, you will be asked to identify the correct modifier. A
modifier can be an adjective or an adjectival phrase that describes a noun or an -
ing form. A modifier can also be an adverb or an adverbial phrase that adds
information about the verb, adjective, or another verb. Adjectives do not change
form to agree with the nouns or -ing forms that they describe, but some adjectives
are used only with count nouns and others are used only with noncount nouns.
Choose the correct answer in the incomplete sentences. Choose the incorrect word or
phrase in the underlined choices.

1. The data on the winter migration patterns of the monarch butterfly is very ____
(A) interested
(B) interest
(C) interesting
(D) of interest
2. There are more potatoes cultivated than any the other vegetable crop worldwide.
(A) (B) (C) (D)
3. Marian Anderson, recognized both in the U.S. and in Europe as a real great vocalist,
(A) (B)
was the first black singer to appear with the Metropolitan Opera Company.
(C) (D)
4. The New England states have had _____ serious earthquakes since the Ice Age.
(A) none
(B) not any
(C) not
( D ) no
5. ___ orangutans live alone.
(A) Near all
(B) Almost all
(C) The all
(D) The most all
6. Some hybrid flowers retain the fragrant scent of the nonhybrid, and ______ are
bred without fragrance.
(A) anothers
(B) the other
(C) some other
(D) others
7. At the core of a star, temperatures and pressures are so great as particles collide and
(A) (B)
connect in a process called fusion.
(C) (D)
8. The Cartwheel Galaxy is 500 million light year away from Earth.
(A) (B) (C) (D)

60
TOEFL Preparation Exercises Structure and Written Expression
9. According to a recent survey, __ doctors do not have a personal physician.
( A ) a l ar g e a mo u nt o f
( B) la rg e a mou nt of
( C ) a l a r g e nu mb e r o f
(D) large number of
10. Because none of food is as nutritious for a baby as its mother's milk, many women
(A) (B) (C) (D)
are returning to the practice of breast feeding.
11. John F. Kennedy was the youngest president of the United States, and _______
to be assassinated.
(A) the fourth
(B) fourth
(C) four
(D) the four
12. Euthanasia, the practice of assisting the death of a person suffering from an
(A)
incurable disease, is such a controversial issue as it is illegal in most countries.
(B)(C) (D)
13. __ in the world export diamonds.
(A) Only little nations
(B) Only few nations
(C) Only a little nations
(D) Only a few nations

14. Uranus is just ________ to be seen on a clear night with the naked eye.
(A) bright enough
(B) enough brightly
(C) as enough bright
(D) bright as enough
15. The conversations on the TOEFL will be spoken just one time; therefore, you must
(A)
listen very careful in order to understand what the speakers have said.
(B) (C) (D)
16. Gold, silver, and copper coins are often alloyed with harder metals to make them
(A) (B) (C)
hard as enough to withstand wear.
(D)

61
TOEFL Preparation Exercises Structure and Written Expression
17. __ like McDonald’s and Kentucky Fried Chicken have used franchising to
extend their sales internationally.
(D) Chain’s restaurants
(B) Chains restaurants
(C) Chain restaurant
(D) Chain restaurants
18. Thirty-six years after his first flight, at the age of 77, John Glenn proved that
he was not ______ to return to his role as an astronaut.
(A) so old
(B) too old
(C) oldest
(D) very older

19. ________ that is known as Art Deco culminated in the exhibits and
expositions at the World’s Fair in 1939.
(A) The art
(B) Arts
(C) An art
(D) Artist

20. The brightest body in the constellation Hydra, Alphard is only ___________.

(A) a second-magnitude star


(B) a magnitude second star
(C) a star of the magnitude second
(D) a second magnitudes star

62
TOEFL Preparation Exercises Structure and Written Expression
EXERCISE 6: Sentences—Comparatives
In some sentences in the Structure Section on the Paper-Based TOEFL or the
Computer-Based TOEFL, you will be asked to identify the correct comparative. A
comparative can be a word or phrase that expresses similarity or difference. A
comparative can also be a word ending like -er or -est that expresses a degree of
comparison with adjectives and adverbs. Choose the correct answer in the incom-
plete sentences. Choose the incorrect word or phrase in the underlined choices.

1. Tuition at an American university runs ______ twenty thousand dollars a


semester.
(A) so high as
(B) as high to
(C) as high as
(D) as high than
2. Alligators are about the same color than crocodiles, although the adults may be
(A) (B) (C)
slightly darker with broader heads and blunter noses.
(D)
3. Laser discs provide images of best quality than those of either television signals or
(A) (B) (C) (D)
video tapes.
4. The cost of a thirty-second commercial on a network television station is ______
for most businesses.
(A) so much
(B) much
(C) very much
(D) much too much
5. The New York City subway system is the most longest underground railroad
(A) (B) (C)
operating in the world.
(D)
6. School children in the same grade in American schools are usually the same old as
(A) (B) (C) (D)
their classmates.

7. The seed heads of teasel plants raise the nap on coarse tweed cloth than do the
machine tools invented to replace them.
(A) more efficiently
(B) efficiently
(C) more efficient
(D) most efficient

63
TOEFL Preparation Exercises Structure and Written Expression
8. Benjamin Franklin was the editor of the larger newspaper in the colonies,
(A) (B)
a diplomatic representative to France and later to England, and the inventor of many
(C) (D)
useful devices.

9. The standard for cleanliness in the area where a microchip is manufactured is same
(A) (B) (C) (D)
that of an operating room in a hospital.
10. The North American robin is only the European and African robins.
(A) half big
(B) as big half
(C) half as big as
(D) big by half
11. Mountain bikes differ ordinary bicycles in that they have ten or more gears,
(A) (B)
a more rugged frame, and wider treads on the tires.
(C) (D)
12. As a rule, the more rapid the heart rate, faster the pulse.
(A) ( B) ( C) (D)
13. In U.S. law, a misdemeanor is a crime that is a felony, and usually carries a term
of imprisonment of less than one year for most offenses.
(A) lesser than
(B) less severe than
(C) less than severe
(D) severely lesser

14. Although both are mammals, the early stages of development on the part of
placentals differ from ______
(A) marsupials
(B) that of marsupials
(C) those of marsupials
(D) those marsupials
15. Eli Whitney's cotton gin enabled the cotton producers of the early nineteenth
century to increase their production by _________ times the amount produced
prior to the invention.
(A) more fifty
(B) more as fifty
(C) more than fifty
(D) most than fifty

64
TOEFL Preparation Exercises Structure and Written Expression
16. __ 250,000 species of fossils have been discovered in both organized, scientific
searches and by sheer accident.
(A) As much as
(B) As many as
(C) As many
(D) Many as
17. The North's abundance of industry and commercial wealth proved to be a greater
advantage ________ in determining the outcome of the Civil War.
(A) than originally thought
(B) that originally thought
(C) as originally thought
(D) originally thought
18. The Woodstock Music and Art Fair of 1969 captured the essence of the
counterculture movement of the 1960s_____
(A) most than any of other events
(B) best that any other event
(C) than any other events
(D) better than any other event
19. Alike her friend and fellow impressionist artist, Edgar Degas, Mary Cassett used
(A) (B) (C)
brush strokes and colors in new and different ways.
(D)
20. A dancer, while always graceful and precise in her movements, trains _________
any other athlete.
(A) as strenuously
(B) more strenuously as
(C) as strenuously as
(D) as strenuously that

65
TOEFL Preparation Exercises Structure and Written Expression

EXERCISE 7: Sentences—Connectors

In some sentences in the Structure Section of the Paper-Based TOEFL or the


Computer-Based TOEFL, you will be asked to identify the correct connector. A
connector is a word or phrase that joins words, phrases, or clauses. A connector
expresses relationships between the words, phrases, and clauses that it joins. Some
common relationships are cause and result, contradiction, substitution, addition,
exception, example, and purpose. Choose the correct answer in the incomplete
sentences. Choose the incorrect word or phrase in the underlined choices.

1. It is not clear how much students learn _________ television classes without
supervision and monitoring.
(A) for watching
(B)from watching
(C) by watch
(D to watch

2. In spite of the fact that 85 percent of all societies allow the men to take more than
one wife, most prefer monogamy _____ polygamy.
(A) than
(B) to
(C) for
(D) that
3. Some metals such gold, silver, copper, and tin occur naturally, and are easy
(A) (B) (C)
to work.
(D)
4. Stained glass becomes even more beautiful when it because _______ the corrosion
diffuses light.
(A) will age
(B) ages
(C) are aging
(D) aged
5. All of the senses ____ smell must pass through intermediate gateways to be
processed before they are registered in the brain.
(A) until

66
TOEFL Preparation Exercises Structure and Written Expression
(B) but
(C) to
(D) for
6. Because the expense of traditional fuels and the concern that they might run out,
(A) (B) (C)
many countries have been investigating alternative sources of power.
(D)
7. The lights and appliances in most homes use alternating current _______
(A) instead direct current
(B) instead of direct current
(C) that instead direct current
(D) for direct current instead
8. Only seventeen on one hundred business calls get through to the correct person on
(A) (B) (C)
the first attempt.
(D)
9. More murders are reported ______ December in the United States than during any
other month.
(A) on
(B) in
(C) at
(D) for

10. The tendency to develop cancer, even in high-risk individuals, can be decreased
____ the amount of fruit and vegetables in the diet.
(A) to increase
(B) for increase
(C) for increasing
(D) by increasing

11. The concept of lift in aerodynamics refers to the relationship among the increased
(A) (B) (C)
speed of air over the top of a wing and the higher pressure of the slower air
(D)
underneath.

12. If one of the participants in a conversation wonders ________, no real


communication has taken place.
(A) what said the other person
(B) what the other person said

67
TOEFL Preparation Exercises Structure and Written Expression
(C) what did the other person say
(D) what was the other person saying
13. A prism is used to refract light so as it spreads out in a continuous spectrum of colors.
(A) (B) (C) (D)
14. Nuclear power plants are still supported by the society of Professional Engineers
(A) (B)
in spite unfortunate accidents like the one at Three Mile Island.
(C) (D)
15. Neptune is an extremely cold planet, and __________
(A) so does Uranus
(B) so has Uranus
(C) so is Uranus
(D) so Uranus
16. Deserts are often formed _ they are cut off from rain-bearing winds by the
surrounding mountain ranges.
(A) because
(B) in spite of
(C) so
(D) due to
17. There are many beautifully preserved historic buildings ___________
(A) in Beacon Street in Boston
(B) in Beacon Street at Boston
(C) on Beacon Street in Boston
(D) at Beacon Street at Boston
18. ___ the original document, the U.S. Constitution contains ten amendments called
the Bill of Rights.
(A) Beside
(B) Besides
(C) In addition
(D) Also
19. National parks include not only the most scenic places in the nation but places
(A) (B) (C)
distinguished for their historic or scientific interest.
(D)
20. Cooking oil made from corn does not become saturated when heated, and
___________ ______________
(A) neither oil made from soy
(B) oil made from soy does either
(C) neither does oil made from soy
(D) oil made from soy either

68
TOEFL Preparation Exercises Structure and Written Expression

EXERCISE 8: Sentences—Sentences and Clauses


In some sentences in the Structure Section of the Paper-Based TOEFL or the
Computer-Based TOEFL, you will be asked to distinguish between a sentence, also
called a main or independent clause, and a subordinate or dependent clause that is
attached to a sentence. Choose the correct answer in the incomplete sentences. Choose
the incorrect word or phrase in the underlined choices.
1. Some ancient units such as the day, the foot, and the pound, _______ today.
(A) are still in use
(B) that are still in use
(C) which are in use still
(D) still in use
2. Paper money __ by the Continental Congress in order to finance the American
Revolution.
(A) which was issued
(B) was issuing
(C) issued
(D) was issued
3. The plastic arts, mainly sculpture and ceramics, that are produced by modeling or
(A) (B) (C) (D)
molding the materials into interesting shapes.
4. The Scholastic Aptitude Test (SAT) by high school students as a
requirement for admission to many colleges.
(A) which is taken
(B) is taken
(C) taken
(D) is taking
5. Ocean currents that help transfer heat from the equator to the poles, thereby creating
(A) (B) (C)
a more balanced global environment.
(D)
6. Camp David ___ the official country home of the U.S. presidents.
(A) that is
(B) that it is
(C) it is
(D) is
7. Gas and dust that stream away from a comet forming one or more tails that may
(A) (B) (C) (D)
extend for millions of miles.

8. The Northwest Ordinances which regulated the sale and settlement of land between
(A) (B)
the Great Lakes and the Mississippi River, territories still occupied by American
(C) (D)

69
TOEFL Preparation Exercises Structure and Written Expression
Indian nations.

9. ____ considered strong and reliable, and is favored by investors who are
interested in security.
(A) That blue chip stock
(B) Blue chip stock is
(C) It is blue chip stock
(D) Which is blue chip stock
10. Most botanists have observed_____ a period of dormancy, even when conditions
may be favorable for growth.
(A) that seeds exhibiti ng
( B) th at seed s exh ib it
(C) seeds th at exhibiting
(D) seeds that they exhibit
11. La Guardia Airport in New York City _____ for Fiorello La Guardia, one of New
York's most popular mayors.
(A) which is named
(B) named
(C) which named
(D) is named
12. In a meritocracy, intelligence and ability ______ more than social position or
wealth.
(A) which value
(B) that are valued
(C) valuing
(D) are valued

13. The larva of the boll weevil, which it feeds on the immature pods of the cotton plant,
(A) (B) (C)
often destroying an entire crop.
(D)

14. Of all the lawsuits in the world, _____ in U.S. courts.


(A) filed 95 percent of them
(B) 95 percent of them are filed
(C) that filed are 95 percent of them
(D) which of them 95 percent are filed

70
TOEFL Preparation Exercises Structure and Written Expression
15. "Chicago" is a poem ______ in praise of one of the busiest industrial centers in the
U.S.
(A) which by Carl Sandburg
(B) which was written by Carl Sandburg
(C) was written by Carl Sandburg
(D) Carl Sandburg who wrote it

16. __ are kept as pets in almost every country in the world.


(A) Cats and dogs which
(B) Which cats and dogs
(C) Cats and dogs
(D) That cats and dogs
17. By studying the fossils of pollen, which extremely resistant to decay, researchers
(A) (B)
can gain useful information about the vegetation of the past.
(C) (D)
18. The PTA ____ parents and teachers who support the school by fund-raising and
other activities.
(A) it is a group of
(B) that is a group of
(C) which group of
(D) is a group of
19. The attribution of human characteristics to animals or inanimate objects appears in
(A) (B)
the mythologies of many cultures is a literary device called anthropomorphism.
(C) (D)
20. The jet stream ______ usually occurs at about thirty-five to sixty degrees latitude.
(A) a narrow band of wind that
(B) is a narrow band of wind that
(C) a narrow band of wind
(D) it is a narrow band of wind that

71
TOEFL Preparation Exercises Structure and Written Expression

EXERCISE 9: Sentences—Point of View

In some sentences in the Structure Section of the Paper-Based TOEFL or the


Computer-Based TOEFL, you will be asked to identify errors in point of view.
Point of view is the relationship between the verb in the main clause of a sentence
and other verbs, or between the verbs in a sentence and the adverbs that express
time. Choose the correct answer in the incomplete sentences. Choose the incorrect
word or phrase in the underlined choices.
1. Although there are approximately 120 intensive language institutes in the United
(A) (B) (C)
States in 1970, there are more than four times as many now.
(D)
2. Cartographers cannot make an accurate map because the political situation in many
areas changes so rapidly that they were not able to draw the boundaries correctly.
(A) (B) (C) (D)
3. Although Emily Dickinson publishes only three of her verses before she died, today
(A) (B)
there are more than one thousand of her poems printed in many important collections.
(C) (D)
4. Dew usually disappeared by seven o'clock in the morning when the sun comes up.
(A) (B) (C) (D)
5. Before the 1800s, when William Young made different shoes for right and left feet,
shoes _______ on either foot.
(A) can wear
(B) are wearing
(C) could be worn
(D) worn
6. Seven months before the stock market crashed in 1929, President Hoover said that
(A) (B)
the economy of the nation is secure.
(C) (D)
7. In the Middle Ages, the word ‘masterpiece’ referred to a work that _______ by a
journeyman in order to qualify as a master artisan.
(A) completed
(B) is completed
(C) was completed
(D) complete

72
TOEFL Preparation Exercises Structure and Written Expression
8. Most archaeologists agree that humans are living in the area around Philadelphia
(A) (B)
for about twelve thousand years.
(C) (D)
9. Although we once thought that Saturn has only seven rings, we now know that it has
(A) (B)
hundreds of rings extending for thousands of miles.
(C) (D)
10. Before his death in 1943, in an effort to encourage less dependence on one crop by
the South, George Washington Carver ______ for developing hundreds of
industrial uses for peanuts and sweet potatoes.
(A) has responsibility
(B) were responsibility
(C) is responsible
(D) was responsible

11. The Greek historian Herodotus reported that one hundred thousand men
_____for twenty years to build the Great Pyramid at Gizeh.
(A) employ
(B) employed
(C) are employed
(D) were employed

12. In 1975, according to the National Center for Health Statistics, the average life
(A)
expectancy for people born during that year is 72.4 years.
(B) (C) (D)
13. Champlain founded a base at Port Royal in 1605, and builds a fort at Quebec three
(A) (B) (C) (D)
years later.

14. According to the Congressional Record, almost one third of all new laws in 1991
(A) (B)
are passed to celebrate some day, week, or month for a special interest group's
(C) (D)
purposes, such as Music Week.

15. The first significant engagement of the American Revolution occurs


(A) (B)

73
TOEFL Preparation Exercises Structure and Written Expression
on June 17, 1775, and has been referred to as in the Battle of Bunker Hill.
(C) (D)
16. Evolutionary changes in the speech organs probably ______ the development of
language in humanoids.
(A) to contribute
(B) contribute to
(C) contribution to
(D) contributed to
17. Originally, the purpose of a sampler is to record complex stitches so that they could
(A) (B) (C) (D)
be duplicated later.
18. Before he died, Armand Hammer _____ an extraordinarily diverse business
empire, including interests in oil, livestock, cattle, grain, and art.
(A) established
(B) establishing
(C) establishes
(D) establish

19. Many ancient cultures begin their spiritual life by worshipping the Sun.
(A) (B) (C) (D)
20. People under thirty years old cannot remember when ______ without a computer
terminal.
(A) they have to work
(B) they had to work
(C) their working
(D) working

EXERCISE 10: Sentences—Agreement


In some sentences in the Structure Section of the Paper -Based TOEFL or the
Computer-Based TOEFL, you will be asked to identify errors in agreement.
Agreement is the relationship between a subject and verb or between a pronoun
and noun, or between a pronoun and another pronoun. To agree, a subject and verb
must both be singular or both be plural. To agree, a pronoun and the noun or
pronoun to which it refers must both be singular or plural and both be masculine or
feminine or neuter. Choose the correct answer in the incomplete sentences. Choose
the incorrect word or phrase in the underlined choices.

1. Both a term paper and a final exam is often required for a college class.
(A) (B) (C)(D)

74
TOEFL Preparation Exercises Structure and Written Expression
2. The popularity of soccer in the United States were increased significantly by
(A) (B)
the playing of the World Cup in cities throughout the country in 1994.
(C) (D)
3. How many musical notes of the 11,000 tones that the human ear can distinguish
______ in the musical scale?
(A) it is
(B) is it
(C) there are
(D) are there
4. Not one in a hundred seeds develop into a healthy plant, even under laboratory
(A) (B) (C) (D)
conditions.

5. Nine of every ten people in the world ______ in the country in which they were
born.
(A) living
(B) they are living
(C) lives
(D) live
6. Benjamin Franklin strongly objected to the eagle's being chosen as the national bird
(A) (B)
because of their predatory nature.
( C ) ( D )
7. In order to grow well, the Blue Spruce like other pine trees, require a temperate
(A) (B) (C) (D)
climate.
8. Few airports in the United States is as modern as that of Atlanta.
(A) (B) (C) (D)
9. In the ocean,___ more salt in the d e e p e r w a t e r .
(A) is there
(B) it may be
(C) there is
(D ) it is
10. Work on improving industrial disposal methods were begun in the early 1970s,
(A) (B)
shortly after the Clean Air bill was passed b y C o n g r e s s .
(C) (D)
11. The average temperature of rocks on the surface of the earth _____ 55 degrees F.
(A) be
( B) ar e
(C) is
(D) been

75
TOEFL Preparation Exercises Structure and Written Expression
12. The officials of the Board of Elections ask that each voter present their registration
(A) (B)
car and a valid Texas driver's license before receiving a ballot.
(C) (D)
13. If one has a special medical condition such as diabetes, epilepsy, or allergy, it is
(A)
advisable that they carry identification in order to avoid being given improper
(B) (C) (D)
medication in an emergency.
14. A large percentage of federal employees are participating in an experimental
(A)
four-day work week aimed at curbing gasoline consumption and pollution, two of
(B) (C)
the most urgent problems facing cities today.
(D)
15. A mature grove of Aspen trees often ______ that supports numerous trunks.
(A) have a single system of roots
(B) has a single root system
(C) make a single system from roots
(D) making a single roots system
16. One-cent coins issued in the United States since 1982 is 96 percent zinc.
(A) (B) (C) (D)
17. According to a team of scientists, there are evidence that Mount Everest is still
(A) (B) (C)
rising.
(D)
18. The urinary system, including both the bladder and the kidneys, are contained in
(A) (B) (C) (D)
the cavities of the trunk.

19. The smallest flying dinosaurs ________ of a robin.


(A) about the size
(B) was about the size
(C) were about the size
(D) have been about the size
20. In the 1920s, Art Deco, known for plastic and chrome-plated objects, were very
(A) (B) (C) (D)
popular.

76
TOEFL Preparation Exercises Structure and Written Expression

EXERCISE 11: Sentences—Introductory Verbal Modifiers


In some sentences in the Structure Section of the Paper -Based TOEFL or the
Computer-Based TOEFL, you will be asked to identify errors in introductory verbal
modifiers and the subjects that they modify. Introductory verbal modifiers are -ing
forms, participles, and infinitives. A phrase with an introductory verbal modifier occurs
at the beginning of a sentence and is followed by a comma. The subject modified by an
introductory verbal modifier must follow the comma. If the correct subject does
not follow the comma, then the meaning of the sentence is changed. Often the
changed meaning is not logical. Choose the correct answer in the incomplete
sentences. Choose the incorrect word or phrase in the underlined choices.

1. After finishing Roots, the one-hundred-year history of an African-American family,


(A) (B)
the Nobel Prize committee awarded author Alex Haley a special citation for literary
(C) (D)
excellence.

2. A competitive sport, gymnasts perform before officials who must use their judgment
(A) (B)
along with their knowledge of the rules to determine the relative skill of each
(C) (D)
participant.

3. To remove stains from permanent press clothing, carefully soaking in cold water
(A) (B)
before washing with a regular detergent.
(C) (D)

4. An abstract painter and pioneer of Surrealism, _______ and symbolic images.


(A) Miro's works are characterized by bright colors
(B) the works of Miro are characterized by bright colors
(C) Miro is famous for works characterized by bright colors
(D) bright colors characterize the works of Miro
5. Found in Tanzania by Mary Leakey, some archeologists estimated that
(A)
the three-million-year-old fossils were the oldest human remains to be discovered.
(B) (C) (D)

6. Originally having been buried in Spain, and later moved to Santo Domingo in
(A) (B) (C)

77
TOEFL Preparation Exercises Structure and Written Expression
the Dominican Republic, Columbus’s final resting place is in Andalusia, Spain.
(D)
7. The largest hotel on Earth, ___________
(A) the MGM Grand has 91 elevators and 5005 rooms
(B) there are 91 elevators and 5005 room! In the MGM Grand
(C) 91 elevators and 5005 rooms are in th MGM Grand
(D) it is the MGM Grand that has 91 elevators and 5005 rooms
8. Written by Neil Simon, New York audiences received the new play enthusiastically at
(A) (B) (C)
the world premiere Saturday evening.
(D)
9. To prevent cavities, dental floss should be used daily after brushing one’s teeth.
(A) (B) (C) (D)
10. While researching the problem of violent crime, the Senate committee’s discovery
(A)
that handguns were used to commit 64 percent of all murders in the United States.
(B) (C) (D)

11. One of the world's greatest rivers, _______


(A) one third of North America is linked by the water of the Mississippi
(B) the Mississippi links one third of North America by water
(C) North America is linked by the Mississippi in one third of the water
(D) the water is linked in North America by one third of the Mississippi
12. After reviewing the curriculum, several significant changes were made by the faculty
(A) (B) (C)
in traditional business programs at Harvard University.
(D)
13. Having hit more home runs in one season than any other player in the history of
(A) (B) (C)
baseball, Mark McGwire's record is famous.
(D)
14. Banned in the U.S., the effect of fluorocarbons continues at a level that could
(A) (B)
eventually damage the ozone layer, and bring about such serious results as high risk
(C) (D)
of skin cancer and global climate changes.
15. Published by Penguin Press almost eighty years ago, ________ offered to the
public.
(A) Ernest Hemingway wrote A Farewell to Arms as the first paperback book
(B) A Farewell to Arms was the first paperback book by Ernest Hemingway that it

78
TOEFL Preparation Exercises Structure and Written Expression
was
(C) Ernest Hemingway's book A Farewell to Arms was the first paperback book
(D) it was A Farewell to Arms that was the first paperback by Ernest Hemingway
16. To avoid jet lag, many doctors recommend that their patients begin adjusting one
(A) (B)
week before departure time by shifting one hour each day toward the new time
(C) (D)
schedule.

17. Traditionally named for women, Bob was chosen as the first male name for a
(A) (B) (C) (D)
hurricane.

18. While trying to build a tunnel through the Blue Ridge Mountains, ________
(A) coal was discovered by workmen at the construction site
(B) workmen discovered coal at the construction site
(C) the construction site was where coal was discovered by workmen
(D) it was the construction site where workmen discovered coal
19. Born in 1892, _______ while he wrote the poems and plays that made him
famous.
(A) the Library of Congress is where Archibald MacLeish worked as a librarian
(B) Archibald MacLeish worked as a librarian at the Library of Congress
(C) a librarian at the Library of Congress, Archibald MacLeish worked
(D) at the Library of Congress, Archibald MacLeish worked as a librarian
20. Founded in 1919, students and teachers who are interested in spending several
(A) (B)
months abroad may benefit from educational programs administered by the Institute
(C) (D)
for International Education.

79
TOEFL Preparation Exercises Structure and Written Expression
EXERCISE 12: Sentences—Parallel Structure
In some sentences in the Structure Section of the Paper -Based TOEFL or the
Computer-Based TOEFL, you will be asked to identify errors in parallel structure.
Parallel structure is the use of the same grammatical structures for related ideas of
equal importance. Related ideas of equal importance often occur in the form of a
list. Sometimes related ideas of equal importance are connected by conjunctions,
such as and, but, and or. Choose the correct answer in the incomplete sentences.
Choose the incorrect word or phrase in the underlined choices.

1. Rock music is not only popular in the United States but also abroad.
(A) (B) (C) (D)
2. To control quality and making decisions about production are among the many
(A) (B) (C) (D)
responsibilities of an industrial engineer.
3. Most of the Cajun French who live in Louisiana can neither read ____ the French
variety that they speak fluently.
(A) nor they write
(B) nor write
(C) or writing
(D) neither write
4. The six main parts of a business letter are the address, the inside address,
(A) (B) (C)
the salutation, the body, the closing, and signing your name.
(D)
5. Microwaves are used for cooking, telecommunications, and _________
(A) to diagnose medically
(B) medical diagnosing
(C) diagnosed medically
(D) medical diagnosis
6. To read literature and being introduced to a different culture are two excellent
(A) (B) (C)
reasons for studying a foreign language.
(D)
7. Ice skating and to go skiing are popular winter sports in the northern United States.
(A) (B) (C) (D)
8. To treat minor diarrhea, drink plenty of liquids, especially tea, water, and carbonated
(A)
beverages, eat soup, yogurt, salty cracker; and bananas, and avoiding milk, butter,
(B) (C)
eggs, and meat for twenty-four hours.
(D)

80
TOEFL Preparation Exercises Structure and Written Expression
9. A vacuum will neither conduct heat nor _________
(A) transmit sound waves
(B) transmitting sound waves
(C) sound waves are transmitted
(D) the transmission of sound waves
10. The Smithsonian Institute is famous because it contains such interesting exhibits as
the flag that was raised over Fort McHenry in 1812, the airplane that the Wright
(A) (B)
brothers built for their first flight at Kitty Hawk, and there are the gowns worn
(C) (D)
by every first lady since Martha Washington.

11. In order to become a law, a bill must be passed not only by the Senate but also
(A) (B) (C)
the House of Representatives.
(D)
12. The color of a star depends on the heat and __________
(A) how much energy produced
(B) the energy it produces
(C) production of the energy
(D) producing energy
13. The cloverleaf is a common engineering design for expressways that permits
(A)
traffic between two intersecting highways to move more safely, efficiently,
(B) (C)
and with ease.
(D)
14. A new product should be judged not by the promises made in commercials and
(A) (B)
advertisements, but also by the results demonstrated in actual use.
(C) (D)
15. The artisans of the southwestern United States are famous for their beautiful art
work, especially handmade jewelry cast from silver, carved from stones, or
_______ with beads and feathers.
(A) decorations
(B) decorating
(C) decorated
(D) decorate
16. Snakes stick out their tongues, move them around, and also they retract them
(A) (B) (C)
quickly to pick up odor molecules that aid in detecting direction.
(D)

81
TOEFL Preparation Exercises Structure and Written Expression
17. Thought by some to be the first labor party, the Workingman's Party struggled not
(A) (B)
only for better working conditions also for public schools for all children.
(C) (D)
18. The cerebellum's main functions are the maintenance of posture and move the body.
(A) (B) (C) (D)

19. The Cabinet consists of secretaries of departments who report to the president, give
him advice, and _____ decisions.
(A) helping him making
(B) helping him make
(C) help him making
(D) help him make
20. Increasing involvement in agriculture by large corporations has resulted in what
(A) (B)
is known as agribusiness—that is, agriculture with business techniques, including
(C)
heavy capitalization, specialization of production, and to control all stages of
(D)
the operation

EXERCISE 13: Sentences—Redundancy

In some sentences in the Structure Section of the Paper-Based TOEFL or the


Computer-Based TOEFL, you will be asked to identify errors in redundancy.
Redundancy is the unnecessary repetition of words and phrases. Choose the correct
answer in the incomplete sentences. Choose the incorrect word or phrase in the
underlined choices.

1. Some international students use a cassette recorder to make tapes of their classes
(A) (B)
so that they can repeat the lectures again.
(C) (D)
2. Blood plasma it is the transportation system for all of the widely separated organs
(A) (B) (C)
in the human body.
(D)

3. Whereas a gas expands in all directions, a vapor remains somewhat more


concentrated.
(A) in a uniform manner

82
TOEFL Preparation Exercises Structure and Written Expression
(B) uniformly
(C) uniformly in manner
(D) uniform
4. Appointed by the General Assembly for five years, the Secretary-General of
(A) (B)
the United Nations must act in an impartial manner toward all members.
(C) (D)
5. Humans who lived thousands of years ago, long before alphabets were devised, they
(A) (B) (C)
used pictures to record events and to communicate ideas.
(D)
6. If one does not pick up the dry cleaning within thirty days, the management is not
(A) (B)
obligated to return it back.
(C) (D)
7. That witches cause disasters and misfortunes _______ among the colonists I Salem,
Massachusetts.
(A) it was widely believed
(B) was widely believed
(C) was believed in a wide way
(D) they widely believed
8. The southern part of the United States has ideal conditions for raising cotton
(A)
because the climate is sufficiently warm enough to allow
(B) (C)
a six-month growing period.
(D)
9. People who are competitive in nature are more likely to suffer from the effects of
(A) (B) (C)
stress on their health.
(D)
10. International law is made up of the rules customs that they deal with
(A) (B) (C)
the relationships between different nations and the citizens different nations.
(D)
11. Found in and near the Mohave Desert, ______ has a limited habitat.
(A) is the Joshua tree that it
(B) it is the Joshua tree
(C) the Joshua tree
(D) the Joshua tree it

12. Traditionally, the South has been mostly Democrat ___, while the North has bee
divided between Democrats and Republicans.
83
TOEFL Preparation Exercises Structure and Written Expression
(A) in the politics
(B) politically
(C) politics-wise
(D) in a political way .

13. It was Isadora Duncan who was responsible for many of the new innovations that
(A) (B) (C)
have made modern dance different from classical ballet.
(D)
14. Little House on the Prairie, a successful television program, was adapted from a
series of books by young pioneer woman whose life was similar to that of
(A) (B) (C)
the character called by name Laura.
(D)

15. In recent years great advances forward have been made in the field of
(A) (B) (C)
genetic research.
(D)

16.Today the United States is one of the few countries in the western Hemisphere that
(A)
it has laws providing for the death penalty.
(B) (C) (D)
17. According to recent geological research, the climate of the states along the Canadian
(A) (B)
border is changing with rapidity.
(C) (D)
18. Digital clocks, however precise, _______ because the earth's rotation changes
slightly over the year.
(A) they cannot be perfectly accurate
(B) cannot be perfectly accurate
(C) not perfectly accurate
(D) not be perfectly accurate
19. Natural gas often occurs ________ petroleum in the minute pores of rocks such as
sandstone and limestone.
(A) both together with
(B) both together
(C) with
(D) both with
20. World hunger it is one of the most urgent problems that we face today.
(A) (B) (C) (D)

84
TOEFL Preparation Exercises Structure and Written Expression
EXERCISE 14: Sentences—Word Choice
In some sentences in the Structure Section of the Paper-Based TOEFL or the
Computer-Based TOEFL, you will be asked to identify errors in word choice. Word
choice is the selection of words that express the exact meaning of an idea. Sometimes it
is necessary to make a choice between words that are very similar in appearance but
very different in meaning. Choose the correct answer in the incomplete sentences.
Choose the incorrect word or phrase in the underlined choices.

1. According to the Pythagorean theorem, the sum of the squares of the two sides
(A)
of a triangle is equal as the square of the hypotenuse.
(B) (C) (D)
2. The flag over the White House is risen at dawn every day by a color guard from the
(A) (B) (C) (D)
United States armed forces.
3. Commercials on the educational television network are generally shorter
(A) (B)
comparing those on other networks.
(C) (D)
4. The Pilgrims_______ seven thousand dollars at 43 percent interest to make their
journey in 1620.
(A) lent
(B) borrowing
(C) to lend
(D) borrowed
5. The Food and Drug Administration does not declare a drug a carcinogen until it has
(A)
been proven conclusively that the effects in rats can be generalized for human beings.
(B) (C) (D)
6. In some states, the law allows drivers to turn right at a red light, but in other
(A) (B)
states, the law does not leave them do it.
(C) (D)
7. The effective of a project on the general population is difficult to measure unless
(A) (B)
a statistician is employed to tabulate the variables.
(C) (D)
8. When a person is arrested, the cops must let him make one telephone call.
(A) (B) (C) (D)
9. Although blood _______ in urine and stool samples, it cannot always be detected
without the aid of a microscope.
(A) lets residue
(B) leaves residue
85
TOEFL Preparation Exercises Structure and Written Expression
(C) residues
(D) making residue

10. The audible range of frequencies for human beings _______ between 20 and
20,000 Hz.
(A) lies
(B) lays
(C) lying
(D) laying
11. If the owner of a bar suspicions that someone's identification is not valid, he can
(A) (B)
refuse to serve the order.
(C) (D)
12. The condition of menkind has been improved by recent technological advances.
(A) (B) (C) (D)
13. __________ mammals, once weaned, do not routinely drink milk.
(A) As a whole,
(B) As whole,
(C) Wholly,
(D) On a whole,
14. The classify of plants begins with those having the simplest structure, and
(A) (B)
progresses to include the most highly organized forms in four divisions called
(C) (D)
phylums.

15. With the develop of a cheap process for desalination, 97 percent of the Earth's water
(A) (B) (C)
will become available for freshwater purposes.
(D)
16. People with exceptionally high intelligence quotients may not be the best employees
since they _____ unless the job is constantly changing.
(A) become bored of work
(B) are becoming boring in work
(C) become bored with their work
(D) work becoming bored
17. An understand of calculus is essential to the study of engineering.
(A) (B) (C) (D)
18. Henry Wadsworth Longfellow not only wrote poems and stories but also presided
(A) (B)
the modern language department at Harvard University for more than eighteen
(C) (D)
years.
86
TOEFL Preparation Exercises Structure and Written Expression

19. In cold weather, growers place wind machines ______ the groves to keep the air
circulating and to warm up the citrus crops.
(A) near to
(B) near of
(C) next to
(D) nearly
20. Almost all life depends to chemical reactions with oxygen to produce energy.
(A) (B) (C) (D)

EXERCISE 15: Sentences—Comprehensive Structures


In the Structure Section of the Paper-Based TOEFL, the items will be
organized into two parts—completion sentences and correction sentences. On the
Computer-Based TOEFL, the items will be presented at random. Both of the TOEFL
tests will include a comprehensive selection of structures. This exercise is the
Paper-Based TOEFL format. Choose the correct answer in the incomplete
sentences. Choose the incorrect word or phrase in the underlined choices.

Part 1
1. In simple animals, ________ reflex movement or involuntary response to stimuli.
(A) behavior mostly
(B) most is behavior
(C) most behavior is
(D) the most behavior
2. Although the weather in Martha’s Vineyard isn’t _______ to have a year-round
tourist season, it has become a favorite summer resort.
(A) goodly enough
(B) good enough
(C) good as enough
(D) enough good
3. According to the wave theory, _______ population of the Americas may have been
the result of a number of separate migrations.
(A) the
(B) their
(C) that
(D) whose
4. It is presumed that rules governing the sharing of food influenced _______ that the
earliest culture evolved.

87
TOEFL Preparation Exercises Structure and Written Expression
(A) that the way
(B) is the way
(C) the way
(D) which way
5. Calculus, _______ elegant and economical symbolic system, can reduce complex
problems to simple terms.
(A) it is an
(B) that an
(C) an
(D) is an

6. Canada does not require that US citizens obtain passport to enter the country and
____________
(A) Mexico does neither
(B) Mexico doesn’t either
(C) neither Mexico does
(D) either does Mexico
7. The poet ______ just beginning to be recognized as an important influence at the
time of his death.
(A) being Walt Whitman
(B) who was Walt Whitman
(C) Walt Whitman
(D) Walt Whitman was
8. ________ the formation of the Sun, the planets, and other stars began with the
condensation of an interstellar cloud.
(A) It accepted that
(B) Accepted that
(C) It is accepted that
(D) That is accepted
9. As a general rule, the standard of living ________ by the average output of each
person in society.
(A) is fixed
(B) fixed
(C) has fixed
(D) fixes
10. The Consumer Price Index lists ____________
(A) how much costs every car
(B) how much does every car cost
(C) how much every car costs
(D) how much are every car cost

11. The Ford Theater where Lincoln was shot __________


(A) must restore
88
TOEFL Preparation Exercises Structure and Written Expression
(B) must be restoring
(C) must have been restored
(D) must restored
12. Fast-food restaurants have become popular use many working people want
_________
(A) to eat quickly and cheaply
(B) eating quickly and cheaply
(C) eat quickly and cheaply
(D) the eat quickly and cheaply

13. After seeing the movie Centennial, ____________


(A) the book was read by many people
(B) the book made many people want to read it
(C) many people wanted to read the book
(D) the reading of the book interested many people
14. ___ , Carl Sandburg is also well-known for his multivolume
biography of Lincoln.
(A) An eminent American poet
(B) He is an eminent American poet
(C) An eminent American poet who is
(D) Despite an eminent American poet
15. The examiner made us _______ our identification in order to be admitted to the
test center.
(A) showing
(B) show
(C) showed
(D) to show
16. A swarm of locusts is responsible the consumption of enough plant material to feed
(A) (C) (D)
a million and a half people.
(D)

17. Oyster farming has been practice in most parts of the world for many years.
(A) (B) (C) (D)
18. Those of us who smoke should have their lungs x-rayed regularly.
(A) (B) (C) (D)
19. After the team of geologists had drawn diagrams in their notebooks and wrote
(A) (B)
explanations of the formations which they observed, they returned to their campsite
(C)
to compare notes.
(D)
89
TOEFL Preparation Exercises Structure and Written Expression
20. If Robert Kennedy would have lived a little longer, he probably would have won
(A) (B) (C) (D)
the election.
21. It was Shirley Temple Black which represented her country in the United Nations
(A) (B) (C)
and later became an ambassador.
(D)
22. The prices at chain stores are as reasonable, if not more reasonable, as those at
(A) (B) (C) (D)
discount stores.

23. It is extremely important for an engineer to know to use a computer.


(A) (B) (C) (D)
24. Historically there has been only two major factions in the Republican Party
(A) (B) (C) (D)
—the liberals and the conservatives.

25. Whitman wrote Leaves of Grass as a tribute to the Civil War soldiers who had laid
(A)
on the battlefields and whom he had seen while serving as an army nurse.
(B) (C) (D)
26. One of the first and ultimately the most important purposeful of a reservoir is
(A) (B)
to control flooding.
(C) (D)
27. The Chinese were the first and large ethnic group to work on the construction of the
(A) (B) (C) (D)
transcontinental railroad system.

28. The range of plant life on a mountainside is a results of differences in temperature


(A) (B) (C)
and precipitation at varying altitudes.
(D)
29. Even a professional psychologist may have difficulty talking calm and logically
(A)
about his own problems.

30. The more the relative humidity reading rises, the worst the heat affects us.
(A) (B) (C) (D)
31. Because correlations are not causes, statistical data which are extremely easy
(A) (B) (C)
to misuse.
(D)
90
TOEFL Preparation Exercises Structure and Written Expression
32. Lectures for the week of March 22-26 will include the following: The Causes of the
(A) (B) (C)
Civil War, The Economy of the South, Battle Strategies, and The Assassinate Lincoln.
(D)
33. Despite of many attempts to introduce a universal language, notably Esperanto and
(A) (B) (C)
Idiom Neutral, the effort has met with very little success.
(D)
34. As every other nation, the United States used to define its unit of currency,
(A) (B) (C) (D)
the dollar, in terms of the gold standard.

35. It is necessary that one met with a judge before signing the final papers
(A) (B) (C)
for a divorce.
(D)
36. Until recently, women were forbidden by law from owning property.
(A) (B) (C) (D)
37. According to the graduate catalog, student housing is more cheaper than housing
(A) (B) (C) (D)
off campus.

38. John Dewey thought that children will learn better through participating in
(A) (B)
experiences rather than through listening to lectures.
(C) (D)
39. In England as early as the twelfth century, young boys enjoyed to play football.
(A) (B) (C) (D)
40. Some methods to prevent soil erosion are plowing parallel with the slopes of hills,
(A) (B) (C)
to plant trees on unproductive land, and rotating crops.
(D)

91
Team of TOEFL Course

Arina Shofiya
Puspita Maya Ratri
Muhammad Basuni
Nanik Sri Rahayu

Printed by Language Center


State Islamic University of SATU Tulungagung

ii
Acknowledgement

In the name of Allah the most beneficent and most merciful.


Let's pray to Allah SWT (glory be to Him, and He is the High) to bless Prophet
Muhammad SAW (peace be upon him) and his family. Further, may Allah SWT bless
the Khulafaa' Rashidoon and the Sahaba of the Prophet as well as the Followers and
the Followers of the Followers till the day of judgment.
Considering the need of English in future, State Islamic University of Sayyid Ali
Rahmatullah Tulungagung is trying its best effort to advance the students’ language
competence . One of the programs having served to the students is TOEFL course.
With this program, it is highly expected that the students will be able to achieve
good TOEFL score at the level of above 500. As the result, finally, they will be able to
continue their study to the higher level abroad easily. Above all, as being expected,
they will be able to gain successful life without pain in this competitive era.
Finally, I would like to express my deepest gratitude to the writers and those who
are involved in this work. May Allah SWT bless and reward their good deed. Amen.

Tulungagung, 01 October 2022

Prof.Dr. H. Maftukhin, M.Ag


Rector of State Islamic University of SATU Tulungagung

iii
Preface

In the name of Allah the most beneficent and most merciful.

The author wishes to thank all those colleagues who contributed in making this book
available to the readers.
Special thanks to Dr. Arina Shofiya and her colleagues in the Language
Center, State Islamic University of Sayyid Ali Rahmatullah Tulungagung for their
tremendous help in publishing this new book for the students of State Islamic
University of Sayyid Ali Rahmatullah Tulungagung. May Allah bless and reward
them.
Thanks and appreciation go to Prof. Dr. H. Maftukhin, M.Ag, the Rector of
State Islamic University of Sayyid Ali Rahmatullah Tulungagung, and unforgettably
Dr. H. Muhtadi Anshor, M.Ag, the Vice Rector for Academic Affairs. Without them,
this publication may not be possible for the students. May Allah bless them. Amen.
Last but not least, may Allah SWT bless and love all those who contributed
their time, effort, energy, wealth, wisdom, goodwill, editing, typing, proofreading
and their Du'a.

Tulungagung, 01 October 2022

The Team

iv
Table of Content

Acknowledgement …………………………………………………………………………….. ii
Preface ………………………………….…………………………………………………………… iii
Table of Content ………………….……………………………………………………………. v

Structure and Written Expression


Exercise 1 ……………………………………………………………………………………………….. 57
Exercise 2 ……………………………………………………………………………………………….. 60
Exercise 3 ……………………………………………………………………………………………….. 63
Exercise 4 ……………………………………………………………………………………………….. 66
Exercise 5 ……………………………………………………………………………………………….. 69
Exercise 6 ……………………………………………………………………………………………..…… 72
Exercise 7 ……………………………………………………………………………………………….. 75
Exercise 8 ……………………………………………………………………………………………….. 78
Exercise 9 ……………………………………………………………………………………………….. 81
Exercise 10 ………………………………………………………………………………………….….. 84
Exercise 11 ………………………………………………………………………………………….….. 87
Exercise 12 ..………………………………………………………………………………………….. 90
Exercise 13 ………………………………………………………………………………………………. 93
Exercise 14 ………………………………………………………………………………………………. 96
Exercise 15 ………………………………………………………………………………………………. 99

Reading Comprehension and Vocabulary


Vocabulary Skill…………………………………………………………………………………………. 105
Reading Skill……..……………………………………………………………………………………. 113

v
vi
TOEFL Preparation Exercises Reading Comprehension & Vocabulary
VOCABULARY SKILLS

Each vocabulary question consists of a sentence with an underlined word or group of


words. This sentence is followed by four answer choices. You must choose the answer
that is closest in meaning to the underlined word or group of words. You must also be
sure not to change the meaning of the sentence.

Example

The earth is divided into two hemispheres.


(A) chopped
(B) joined
(C) separated
(D) mixed

Of the four answer choices, answer (C) separated is closest in meaning to the
underlined word divided. Also, the sentence The earth is separated into two
hemispheres has approximately the same meaning as The earth is divided into two
hemispheres. Therefore, answer (C) is the best answer.
There are four ideas that you should remember while you are working on the
vocabulary questions. These ideas will help you plan the best strategies for the
vocabulary questions:
1. The questions progress from easy to difficult.
2. The vocabulary questions do not test grammar.
3. The context does not help very much and in fact is sometimes confusing.
4. You must be careful of second meanings of words.
Each of these ideas will be explained in detail in the following section.
1. The questions progress from easy to difficult.
Of the 30 questions in the Vocabulary Section, the first few questions are rather easy.
The questions become progressively harder throughout the section, and the last few
questions are extremely difficult. Unless your vocabulary is very strong, you should
not expect to know the meanings of the last vocabulary words. You should answer
each question (even if you do not know the meaning of a word) because there is no
penalty for guessing on the TOEFL. However, do not make the mistake of spending
too much time on the vocabulary, particularly on words that you do not know. If you
spend a lot of time on the really difficult vocabulary questions, you will probably not
get too many correct. Worse than that, you will use up valuable time that could be put

105
TOEFL Preparation Exercises Reading Comprehension & Vocabulary
to better use on the reading questions.
2. The vocabulary questions do not test grammar.
In the vocabulary questions the answer choices will always be the same part of speech
as the underlined word or group of words. If the underlined word is a noun, then the
four answer choices will all be nouns; if the underlined word is a verb, then the four
answer choices will all be verbs. You will not have any questions in which you must
choose between different parts of speech. The following example is a typical TOEFL
vocabulary question.
Disneyland first (an adverb) opened for business in 1955.
(A) later (an adverb)
(B) originally (an adverb)
(C) next (an adverb)
(D) finally (an adverb)

In this example, notice that the underlined word first is an adverb and each of the
answer choices are adverbs. The word that is closest in meaning to first is originally.
Therefore, you should choose answer (B).
The next example shows what you will not see in the Vocabulary section of the
TOEFL. You will not have any questions in which you must choose between different
parts of speech.
An Example Of What You Will Not Find On The TOEFL
The shop was having financial difficulties (a noun) because of its huge inventory.
(A) worsen (a verb)
(B) problems (a noun)
(C) serious (an adjective)
(D) terribly (an adverb)

You will not find any examples in the Vocabulary section of the TOEFL in which you
must choose a correct answer because it is the same part of speech as the underlined
word. Therefore, do not waste your time looking at the grammatical construction of
the sentences in the Vocabulary section.

3. The context does not help much.


The context of the TOEFL vocabulary questions does not help you very much to
understand the underlined vocabulary word. In fact, the context is often designed to
lead you to an incorrect answer.

106
TOEFL Preparation Exercises Reading Comprehension & Vocabulary
Example

In the middle of the desert, they found a refreshing spring.


(A) hot
(B) dry
(C) sandy
(D) cooling
In this example, if you do not know the meaning of refreshing, you might look at the
context and see the word desert. Since a desert is hot, dry, and sandy, you might
choose answers (A), (B), or (C). However, the answer that is closest in meaning to
refreshing is answer (D) cooling. Therefore, you should choose answer (D). You
should not let yourself be confused by the mention of desert in the context around the
underlined vocabulary word.

4. Beware of second meanings.


Words in English often have more than one meaning. The word embrace, for
example, has a primary meaning of hug, but it can also mean include. In the
Vocabulary section of the TOEFL, the primary meaning of a word can be an
incorrect answer and a secondary meaning of that word can be the correct
answer. (In this case, the immediate context can be of some help in determining
which of several possible meanings is correct.)

Example

His essay embraced all the main points of his philosophy.


(A) hugged
(B) entertained
(C) interlaced
(D) included
Although embraced in some sentences can mean hugged, in this sentence
it means included. Therefore, the best answer is (D).

STRATEGIES FOR THE VOCABULARY QUESTIONS


1. Work quickly through the Vocabulary Section. The questions
progress from easy to difficult. Do not spend too much time on the
difficult questions.
2. Do not look for grammatical clues to help you decide which
vocabulary word goes in the blank. The Vocabulary Section does not
test grammar.

107
TOEFL Preparation Exercises Reading Comprehension & Vocabulary
3. Do not spend a lot of time looking for contextual clues to the
meanings of words. The context usually doesn't help you understand
the meaning of the word.
4. Be careful of secondary meanings of words. Words in English often
have more than one meaning. The word ''embrace,'' for example, has a
primary meaning of ''hug,'' but it can also mean "Include.'' You, must be
aware that in the Vocabulary Section of the TOEFL, the primary meaning
of a word can be an incorrect answer and a secondary meaning of that
word can be the correct answer
5. Never leave any answers blank. Be sure to answer every question in
the
Vocabulary Section even if you do not know the meanings of the words.

EXERCISE 1: You should do this exercise two times

First, do the exercise as quickly as possible. Look only at the underlined word and
a few words around it to determine the context. Do not waste time by studying the
rest of the sentence. Do not look at the grammatical construction of the sentence.
You should complete the ten questions in this exercise in two minutes or less.
Then, look at the exercise a second time, but this time spend all the time that
you want on each question. Study the context. Read and reread each sentence. Do
you change your mind about the answers when you complete the exercise slowly,
or do you choose the same answers regardless of how quickly or slowly you do the
exercise?

1. When viewed from the earth, planets move in a westward direction through the sky.
(A) reflected
(B) seen
(C) reconstructed
(D) envisioned
2. The Appalachian Mountains run parallel to the Atlantic shore, from the Gulf of St.
Lawrence to Alabama.
(A) ocean
(B) Cliffs
(C) islands
(D) coast
3. Your eyes need approximately 20 to 30 minutes to adjust to darkness.

108
TOEFL Preparation Exercises Reading Comprehension & Vocabulary
(A) potentially
(B) appropriately
(C) roughly
(D) exactly
4. Prior to the battle between the ironclad warships Monitor and Merrimack in
1862, warships had been constructed entirely of oak.
(A) upon
(B) Because of
(C) Before
(D) With the advent of
5. Sir Francis Drake circumnavigated the globe from 1577 to 1580.
(A) earth
(B) continent
(C) equator
(D) latitude
6. During the nineteenth century, early chemists discovered more than half of the 100
known elements.
(A) basic
(B) recognized
(C) saluted
(D) met
7. A potato consists of 80 percent water, 17 percent starch, and 3 percent protein.
(A) retains
(B) thrives on
(C) is composed of
(D) is enhanced with
8. Magnolia is a family of trees and shrubs that are native to North America and
Asia.
(A) flowers
(B) leaves
(C) trunks
(D) bushes
9. The Peachtree is native to Georgia.
(A) indigenous
(B) transported

109
TOEFL Preparation Exercises Reading Comprehension & Vocabulary
(C) allocated
(D) imported
10. Pearls come in various colors, but black pearls are generally considered the most
valuable.
(A) designated
(B) deemed
(C) classified as
(D) appointed

When you have completed this exercise, you should begin to decide on a strategy on
the Vocabulary section of the TOEFL. If you have the same number of correct answers
when you work slowly or quickly, you should of course work very quickly in the section
of the TOEFL. If you have more correct answers when you work more slowly, then it is
of course to your advantage to work more slowly in this section of the TOEFL.
However, remember that you need to work fairly quickly on the vocabulary questions
so that you will have enough time for the reading comprehension questions.

EXERCISE 2: Follow the same directions for this exercise that you used in Exercise I.

1. 'The steamship Clermont made a trip 6. About one-third of Canada's people


from New York to Albany on the inhabit the province of Ontario.
Hudson River on August 17, 1807. (A) inherit
(A) voyage (B) reside in
(B) distance (C) depart from
(C) maneuver (D) are born in
(D) vacation 7. A fluorescent lamp uses a much
2. The first U.S. census took 18 months to smaller amount of electricity than an
complete in spite of the fact that there incandescent lamp to produce a
were fewer than 4 million people living similar amount of light.
in the country. (A) comparable
(A) as a result of (B) regular
(B) despite (C) simultaneous
(C) because of (D) gigantic
(D) including 8. The false theory of spontaneous
3. Shirley Chisholm is famous as the first generation was disproved by Louis
black woman to be elected to the U.S. Pasteur.
Congress. (A) sagacious

110
TOEFL Preparation Exercises Reading Comprehension & Vocabulary
(A) disowned (B) circumstantial
(B) renowned (C) automatic
(C) proposed (D) surreptitious
(D) recorded 9. If severe hemorrhaging is not stopped,
4. The stars vary tremendously in size, death can result in minutes
brightness, and temperature. (A) coughing
(A) advantageously (B) trembling
(B) diversely (C) paralysis
(C) , enormously (D) bleeding
(D) creatively 10. After gold was discovered in
5. Emergency food and clothing are California in 1848, the population
needed at the earthquake site. there swelled.
(A) location (A) retracted
(B) center (B) acquiesced
(C) shelter (C) curtailed
(D) damage (D) burgeoned

EXERCISE 3: Follow the same directions for this exercise that you used in Exercise I

1. The Arctic tern breeds in the Arctic regions of North America and then migrates
17,000 kilometers to the waters of Antarctica.
(A) Aspects
(B) Areas
(C) Islands
(D) Marshes

2. Weather satellites can monitor the extent of the earth's snow and ice cover.
(A) check
(B) cut
(C) increase
(D) photograph
3. The Panama Canal, linking the Atlantic and Pacific oceans, was opened in 1014.
(A) joining
(B) closing
(C) crossing
(D) retaining
4. It is believed that the earth's core consists of iron and nickel oxides, whereas the
crust is mainly granite.
(A) while
(B) or

111
TOEFL Preparation Exercises Reading Comprehension & Vocabulary
(C) whenever
(D) whereby
5. Penicillin is widely used to treat bacterial infections.
(A) Transmit
(B) repair
(C) deflect
(D) cure

6. Although fumes still escape from Mount Ranier's volcanic core, the volcano was
largely formed a long time ago.
(A) Hugely
(B) mainly
(C) repeatedly
(D) lengthily
7. The Peloponnesian War was a series of battles between the Greek city-states of
Sparta and Athens.
(A) combination
(B) succession
(C) postponement
(D) forfeiture
8. Like the income statement, the statement of changes in retained earnings refers to a
period of time rather than a single date.
(A) includes
(B) digests
(C) applies to
(D) selects
9. The highest rank in the United States army is that of general.
(A) grade
(B) commander
(C) appointment
(D) executive
10. Because of the moon's weak gravity, it has little or no atmosphere.
(A) potent
(B) feeble
(C) negative
(D) harmless

112
TOEFL Preparation Exercises Reading Comprehension & Vocabulary
READING SKILL

The Reading Comprehension Section of the TOEFL, consists of five or six


reading passages, each followed by four to eight questions. Topics of the
reading passages are varied, but they are often informational subjects that might
be studied in an American university: American history, literature, art, architecture,
geology, geography, and astronomy, for example.

Time is definitely a factor in the Reading section. Many students who take
the TOEFL note that they are unable to finish all the questions in this
section. Therefore, you need to make the most efficient use of your time in this
section to get the highest possible score.
The following method is the best way of attacking a reading passage and
its accompanying questions to get the most questions correct in a limited amount
of time:

STRATEGIES FOR THE READING PASSAGES

1. Skim the reading passage to determine the main idea and the
overall organization of ideas in the passage. You do not need to
understand every passage to answer the questions correctly. It is therefore a
waste of time to read the passage with the intent of understanding every
single detail before you try to answer the questions.
2. Look ahead at the questions to determine what types of
questions you must answer. If you know the types of questions you
must answer, you will know where to look in the passage to find the answers to
the questions.
3. Find the section of the passage that deals with each question. If you
understand the overall organization of ideas in the passage and you know
questions you must answer, you will know exactly where to loo k in the
passage to find the correct answers. You can study the section of the passage
that deals with each question thoroughly and carefully.
4. Choose the best answer to each question from the four answer
choices listed in your test booklet. You can choose the best answer or
answers according to what is given in the appropriate section of the passage,

113
TOEFL Preparation Exercises Reading Comprehension & Vocabulary
eliminate definitely wrong answers, and mark your best guess on the answer
sheet.

The following skills will help you to implement these strategies in the Read ing
section

L O O K I N G A H E A D AT THE READINGS

SKILL 1: RECOGNIZE THE OVERALL ORGANIZATION OF A PASSAGE

The first step when you come to a reading passage is to skim the passage. When you
look ahead at a passage, you are looking for the main idea and the overall organization
of details. TOEFL reading passages are generally very well organized, and the
organization therefore is often easy to spot.
Because time is a factor in the Reading section, you do not have time to read and
reread each passage. You do not have time to read a passage carefully, read the
questions carefully, and then read the passage carefully again to find the answers to the
questions. This is the method many students use, and it is the students who use this
method of rereading the passages who do not have enough time to finish the Reading
section.
Therefore, the first time you look at a reading passage, do not read each
detail carefully. Just look for the main idea and the overall organization
of details. You can probably find the main idea by looking for a topic sentence at the
beginning of each paragraph. You can probably find the overall organization of details
by skimming the passage for clues as to how the details are organized. Most (but not
all) TOEFL passages have a very clear flow of ideas that can be spotted quickly.

Example

There are two very common types of calendars, one determined by the sun
and the other by the moon. The solar calendar is based on the solar year. Since the
solar year is 365.2422 days long, solar calendars consist of regular years of 365
days and have an extra day every fourth year, or leap year, to make up for the
additional fractional amount. In a solar calendar, the waxing and waning of the
moon can take place at various stages of each month. The lunar calendar is
synchronized to the lunar month rather than the solar year. Since the lunar month is
twenty-nine and a half days long, most lunar calendars have alternating months of

114
TOEFL Preparation Exercises Reading Comprehension & Vocabulary
twenty-nine and thirty days. A twelve-month lunar year thus has 354 days, eleven
days shorter than a solar year.

In this example you should start by reading the first sentence carefully because it is
probably the topic sentence. Careful reading of this sentence indicates that this
passage is about two very common types of calendars.
After you find the topic sentence and you understand that the passage is about two
different types of calendars, you should pass your eyes quickly over the rest of the
passage to determine how the details are organized. You are looking for two very
common types of calendars, so it is easy to find the solar calendar (line 2) and the
lunar calendar (line 6). You can therefore quickly recognize that this passage is
organized in the following way:

 A topic sentence discussing two types of calendars


 Three sentences about the solar calendar
 Three sentences about the lunar calendar

EXERCISE 1: Skim each passage looking for the main idea and the organization of
details. (1) Circle the topic sentence(s). (2) Then underline any words that show you
how the details are organized. The first one has been done for you.
PASSAGE ONE
The United States does not have a national university but the idea has been
around for quite some time. George Washington first recommended the idea to
Congress; he even selected an actual site in Washington, D.C., and then left an
endowment for the proposed national university in his will. During the century
following the Revolution the idea of a national university continued to receive
the support of various U.S. presidents, and philanthropist Andrew Carnegie pur-
sued the cause at the beginning of the present century. Although the original
idea has not yet been acted upon, it continues to be proposed in bills before
Congress.

PASSAGE TWO
Within organizations, management may see conflict from one of two con-
trasting points of view. According to the traditional view of conflict, all conflict is
harmful to an organization. Managers with this traditional view of conflict see it
as their role in an organization to rid the organization of any possible sources of
conflict. The interactionist view of conflict, on the other hand, holds that conflict
can serve an important function in an organization by reducing complacency
among workers and causing positive changes to occur. Managers who hold an

115
TOEFL Preparation Exercises Reading Comprehension & Vocabulary
interactionist view of conflict may actually take steps to stimulate conflict within
the organization.

PASSAGE THREE
Chamber music received its name because it was originally intended to be
performed in small rooms in private homes rather than huge concert halls or
theaters. Today it has evolved into small ensemble music in which each per-
former in the ensemble plays an individual part. The compositions written for
this type of performance can easily be classified into three distinct periods, each
with its style of music and instrumentation. In the earliest period (1450-1650),
the viol and other instrumental families developed considerably, and instrumen-
tal music took its first steps toward equal footing with vocal music. In the second
period (1650-1750), trio sonatas dominated. These ensemble compositions were
often written for two violins and a cello; the harpsichord was also featured in
various compositions of this period. In the modern period (after 1750), the pre-
ponderance of chamber music was written for the string quartet, an ensemble
composed of two violins, a viola, and a bass.

PASSAGE FOUR
Vaccines are prepared from harmful viruses or bacteria and administered to
patients to provide immunity to specific diseases. The various types of vaccines
are classified according to the method by which they are derived.

The most basic class of vaccines actually contains disease-causing micro-


organisms that have been killed with a solution containing formaldehyde. In this
type of vaccine, the microorganisms are dead and therefore cannot cause disease;
however, the antigens found in and on the microorganisms can still stimulate the
formation of antibodies. Examples of this type of vaccine are those that fight
influenza, typhoid fever, and cholera.
A second type of vaccine contains the toxins produced by the microorganisms
rather than the microorganisms themselves. This type of vaccine is prepared
when the microorganism itself does little damage but the toxin with the
microorganism is extremely harmful. For example, the bacteria that cause diph-
theria can thrive in the throat without much harm, but when toxins are released
from the bacteria, muscles can become paralyzed and death can ensue.
A final type of vaccine contains living microorganisms that have been ren-
dered harmless. With this type of vaccine, a large number of antigen molecules
are produced and the immunity that results is generally longer lasting than the
immunity from other types of vaccines. The Sabin oral antipolio vaccine and the
BGG vaccine against tuberculosis are examples of this type of vaccine.

116
TOEFL Preparation Exercises Reading Comprehension & Vocabulary
SKILL 2: RECOGNIZE THE DIFFERENT TYPES OF QUESTIONS

There are four types of questions about the reading passages. It is important to
recognize the various types because you will use a different strategy to find the correct
answer to each of them. In this skill you will practice recognizing the various types of
questions and learn strategies for answering them. TOEFL reading questions can be
classified in the following way:

DIRECTLY ANSWERED INDIRECTLY


QUESTIONS ANSWERED
QUESTIONS

ABOUT THE What is the main idea? What is the tone?


WHOLE PASSAGE What is the topic? What is the author's
What is the subject? purpose?
What is the best title? In which course….
The passage suggests
that….

ABOUT PART OF According to the passage…… It can be inferred that….


THE PASSAGE It is indicated that….. It is implied that….
It is stated that…… What most likely
Which is not true….. happened?
What does this word
mean?
What probably came
before/later?

Questions about the main idea, topic, subject, or title are classified as directly
answered in the passage in this classification system. This is because the great
majority of TOEFL passages begin with a topic sentence, so that questions about the
main idea, topic, subject, or title are considered to be directly answered in the topic
sentence.

When you first look at a question in the reading section, you should think about what
type of question it is. You should ask yourself if it is about the whole passage or only
part of the passage. You should ask yourself if it is directly or indirectly answered in
the passage.

117
TOEFL Preparation Exercises Reading Comprehension & Vocabulary
Example

You will read the question:


What is the topic of this passage?

You will ask yourself:


1. Is this question about the whole passage or just one part of the passage? (It is
asking about the whole passage, not just one detail.)
2. Is this question answered directly or indirectly in the passage? (It is probably
answered directly in the main idea or topic sentence of the passage.)

EXERCISE 2: Study each of the following questions from reading passages. First,
mark if the question is about the whole passage or one part of the passage. Then,
mark if the question will probably be answered directly or indirectly in the passage.
Underline any words that help you to answer these questions. The first one has been
done for you.

1. According to the passage, what is the average number of students per class at the
University?
This question is about (a) ____the whole passage
(b) √ part of the passage
This question is answered (c)  directly in the passage
(d) ____indirectly in the passage
2. It can be inferred from the passage that the author approves of which of the
following?
This question is about: (a) — the whole passage
(b)— part of the passage
This question is answered: (c) __directly in the passage
(d)__indirectly in the passage
3. Which of the following best states the main idea of the passage?
This question is about: (a) — the whole passage
(b) __part of the passage
This question is answered (c) — directly in the passage
(d) — indirectly in the passage
4. It is stated in the passage that the sleep cycle of a carnivorous animal is……
This question is about: (a) — the whole passage
(b) __part of the passage

118
TOEFL Preparation Exercises Reading Comprehension & Vocabulary
This question is answered (c) — directly in the passage
(d) — indirectly in the passage

5. The word "adhocracy" (line 2) is closest in meaning to which of the following


words?
This question is about (a) the whole passage
(b) part of the passage
This question is answered (c) directly in the passage
(d) indirectly in the passage
6. What is the author's purpose in this passage?
This question is about (a) ___ the whole passage
(b) ___ part of the passage
This question is answered (c) ___ directly in the passage
(d) ___ indirectly in the passage
7. Which of the following is implied in the passage about the role of women in
factories during the war?
This question is about (a) ___ the whole passage
(b) ___ part of the passage
This question is answered (c) directly in the passage
(d) indirectly in the passage
8. This passage would probably be assigned reading in which course?
This question is about (a) — the whole passage
(b) — part of the passage
This question is answered (c) — directly in the passage
(d) — indirectly in the passage
9. Which of the following best describes the tone of this passage?
This question is about (a) — the whole passage
(b) part of the passage
This question is answered (c) — directly in the passage
(d) — indirectly in the passage
10. The passage indicates that which species is almost extinct?
This question is about (a) — the whole passage
(b) ___ part of the passage
This question is answered (c) directly in the passage
(d) — indirectly in the passage

119
TOEFL Preparation Exercises Reading Comprehension & Vocabulary

ANSWERING DIFFERENT TYPES OF QUESTIONS

SKILL 3: FIND DIRECT ANSWERS ABOUT THE WHOLE PASSAGE

Some questions in the Reading section ask about the passage as a whole rather than -
just one small detail of the passage, and these questions can often be answered by
carefully studying the topic sentence of the paragraph. The following are examples of
this kind of question.

QUESTIONS ABOUT THE TOPIC OF THE PASSAGE

 WHAT IS THE TOPIC OF THIS PASSAGE?


 WHAT IS THE SUBJECT OF THIS PASSAGE?
 WHAT DOES THIS PASSAGE MAINLY DISCUSS?
If a question asks about the topic or subject of a passage, it is asking about what is
discussed throughout the passage.

QUESTIONS ABOUT THE MAIN IDEA OF THE PASSAGE

 WHAT IS THE MAIN IDEA OF THE PASSAGE?


 WHAT IS THE AUTHOR'S MAIN POINT IN THIS PASSAGE?
A question about the main idea of a passage is a little more involved than a question
about the topic. The main idea of a passage is the topic of a passage and the author's
idea about that topic. For example, if the topic of a passage is George Washington's
childhood, then the main idea might be that George Washington's childhood was
happy or that George Washington's childhood had a profound effect on George's later
career.

QUESTIONS ABOUT THE TITLE OF THE PASSAGE

 WHICH OF THE FOLLOWING WOULD BE THE BEST TITLE?


A question about the best title for a passage -is asking almost the same information as
a topic question, but it is written as a title with capital letters. For example, if the topic
of a passage is George Washington's childhood, then the best title for the passage
might be George Washington's Childhood or The Childhood of the First American
President.

120
TOEFL Preparation Exercises Reading Comprehension & Vocabulary

STRATEGIES FOR ANSWERING DIRECT QUESTIONS ABOUT THE WHOLE


PASSAGE

All of these kinds of questions are asking for the same type of information: they are
asking for information about the passage as a whole. When you answer these kinds of
questions, you should remember that (1) information about the whole passage is
generally given in the first sentence (the topic sentence) of each paragraph, and (2)
the incorrect answers are often true details from the passage. The following strategies
work best for questions about the topic, main idea, or title.

STRATEGIES: DIRECT QUESTIONS ABOUT THE WHOLE PASSAGE

1. Read the first sentence of each paragraph carefully to determine the topic and
main idea of the passage.

2. Pass your eyes quickly over the rest of the passage to check that the first
sentence is the topic sentence of the passage.
3. Eliminate any definitely wrong answers and choose the best of the remaining
ones.

Example One
In the philosophy of John Dewey a sharp distinction is made between
"intelligence" and "reasoning." According to Dewey, intelligence is the only
absolute way to achieve a balance between realism and idealism, between
practicality and wisdom of life. Intelligence involves "interacting with other
things and knowing them," while reasoning is merely the act of an
observer, ". . . a mind that beholds or grasps objects outside the world of
things. . . ." With reasoning, a level of mental certainty can be achieved, but
it is through intelligence that control is taken of events that shape one's life.

What is the topic of this passage?


(A) The intelligence of John Dewey
(B) Distinctions made by John Dewey
(C) Dewey's ideas on the ability to reason
(D) How intelligence differs from reasoning in Dewey's works
When you are looking for the topic of a passage with only one paragraph, you
should read the first sentence of the passage carefully because it is probably a
topic sentence. The first sentence of this passage discusses a distinction between
121
TOEFL Preparation Exercises Reading Comprehension & Vocabulary
the ideas of "intelligence" and "reasoning" in the philosophy of John Dewey, so
this is probably the topic. A quick check of the rest of the sentences in the
passage confirms that the topic is in fact the difference between "intelligence"
and "reasoning."
Now you should check each of the answers to determine which one comes
closest to the topic you have determined. Answer (A) mentions only intelligence,
so it is not the topic. Answer (B) is too general: it mentions distinctions that
John Dewey makes, but it does not say specifically what type of distinctions.
Answer (C) mentions only reasoning, when the passage discusses a distinction
between reasoning and intelligence, so answer (C) is incomplete. The best
answer is therefore (D); the idea of how intelligence differs from reason-
ing comes from the first sentence of the passage, which mentions a sharp
distinction........ between "intelligence" and "reasoning."
Example Two
Nitrogen fixation is a process by which additional nitrogen is continuously
fed into biological circulation. In this process, certain algae and bacteria convert
nitrogen into ammonia (NH3). This newly created ammonia is then for the most
part absorbed by plants.
The opposite process of denitrification returns nitrogen to the air. During
the process of denitrification, bacteria cause some of the nitrates from the soil to
convert into gaseous nitrogen or nitrous oxide (N2O). In this gaseous form the
nitrogen returns to the atmosphere.
Which of the following would be the best title for this passage?
(A) The Process of Nitrogen Fixation
(B) Two Nitrogen Processes
(C) The Return of Nitrogen to the Air
(D) The Effects of Nitrogen on Plant Life

In a passage with more than one paragraph, you should be sure to read the first
sentence of each paragraph to determine the subject, title, or main idea. In
Example Two, the first sentence of the first paragraph indicates that the first
paragraph is about the process of nitrogen fixation. If you look only at the first
paragraph, you might choose the incorrect answer (A), which would be a good
title for the first paragraph only. The first sentence of the second paragraph
indicates that the process of denitrification is discussed in the second
paragraph. Answer (C) is incorrect because the return of nitrogen to the
air is the process of denitrification, and this is discussed in the second
paragraph only. Answer (D) is incorrect because the effects of nitrogen on
plant life is not discussed in this passage. The best answer to this question is
answer (B); the two nitrogen processes are nitrogen fixation, which is discussed

122
TOEFL Preparation Exercises Reading Comprehension & Vocabulary
in the first paragraph, and denitrification, which is discussed in the second
paragraph.

EXERCISE 3: Study each of the passages and choose the best answers to the
questions that follow. In this exercise, each paragraph is followed by two main ideas,
topics, or title questions so that the students can practice this type of question. On the
TOEFL one paragraph would probably not have two such questions because they are
so similar.

PASSAGE ONE (Questions 1-2)


Fort Knox, Kentucky, is the site of a U.S. army post, but it is even more
renowned for the Fort Knox Bullion Depository, the massive vault that contains
the bulk of the U.S. government's gold deposits. Completed in 1936, the vault is
housed in a two-story building constructed of granite, steel, and concrete; the
vault itself is made of steel and concrete and has a door that weighs more than
twenty tons. Naturally, the most up-to-date security devices available are in place
at Fort Knox, and the army post nearby provides further protection.
1. Which of the following best describes the topic of the passage?
(A) the city of Fort Knox, Kentucky
(B) the federal gold depository
(C) the U.S. army post at Fort Knox
(D) gold bullion
2. Which of the following would be the best title for this passage?
(A) The Massive Concrete Vault
(B) Fort Knox Security
(C) Where the U.S. keeps its gold
(D) A Visit to Kentucky

PASSAGE TWO (Questions 3-4)


One identifying characteristic of minerals is their re lative hardness,
which can be determined by scratching one mineral with another. In this
type of test, a harder mineral can scratch a softer one, but a softer mineral
is unable to scratch the harder one. The Mohs' hardness scale is used to
rank minerals according to hardness. Ten minerals are listed in this scale,
ranging from talc with a hardness of 1 to diamond with a hardness of 10.
On this scale, quartz (number 7) is harder than feldspar (number 6) and is

123
TOEFL Preparation Exercises Reading Comprehension & Vocabulary
therefore able to scratch it; however, feldspar is unable to make a mark on
quartz.

3. Which of the following best describes the 4. The main idea of this passage is that
subject of this passage? (A) the hardness of a mineral can
(A) The hardness of diamonds be determined by its ability to
(B) Identifying minerals by make a mark on other
means of a scratch test minerals
(C) Feldspar on the Mohs' scale (B) diamonds, with a hardness of 10
(D) Recognizing minerals in on the Mohs' scale, can scratch
their natural state all other minerals
(C) a softer mineral cannot be
scratched by a harder
mineral
(D) talc is the first mineral listed
on the Mohs' scale

PASSAGE THREE (Questions 5-6)


Hurricanes generally occur in the North Atlantic from May through Novem-
ber, with the peak of the hurricane season in September; only rarely will they
occur December through April in that part of the ocean. The main reason for the
occurrence of hurricanes during this period is that the temperature on the
water's surface is at its warmest and the humidity of the air is at its highest.
Of the tropical storms that occur each year in the North Atlantic, only about
five, on the average, are powerful enough to be called hurricanes. To be
classified as a hurricane, a tropical storm must have winds reaching speeds of at
least 117 kilometers per hour, but the winds are often much stronger than that;
the winds of intense hurricanes can easily surpass 240 kilometers per hour.
5. This passage mainly discusses
(A) how many hurricanes occur each year
(B) the strength of hurricanes
(C) the weather in the North Atlantic
(D) hurricanes in one part of the ;world
6. The best title for this passage would be
(A) The North Atlantic Ocean
(B) Storms of the Northern Atlantic
(C) Hurricanes: the Damage and Destruction
(D) What Happens May through November

124
TOEFL Preparation Exercises Reading Comprehension & Vocabulary
PASSAGE FOUR (Questions 7-8)
A hoax, unlike an honest error, is a deliberately concocted plan to present an
untruth as the truth. It can take the form of a fraud, a fake, a swindle, or a
forgery, and can be accomplished in almost any field: successful hoaxes have
been foisted on the public in fields as varied as politics, religion, science, art, and
literature.
A famous scientific hoax occurred in 1912 when Charles Dawson claimed to
have uncovered a human skull and jawbone on the Piltdown Common in
southern England. These human remains were said to be more than 500,000
years old and were unlike any other remains from that period; as such they
represented an important discovery in the study of human evolution. These
remains, popularly known as the Piltdown Man and scientifically named
Eoanthropus dawsoni after their discoverer, confounded scientists for more
than forty years. Finally in 1953 a chemical analysis was used to date the bones,
and it was found that the bones were modern bones that had been skillfully aged.
A further twist to the hoax was that the skull belonged to a human and the jaws
to an orangutan.

7. The topic of this passage could best 8. The author's main point is that
be described as (A) various types of hoaxes have
(A) the Piltdown Man been perpetrated
(B) Charles Dawson's discovery (B) Charles Dawson discovered a
(C) Eoanthropus dawsoni human skull and jawbone
(D) a definition and example of a (C) Charles Dawson was not an
hoax honest man
(D) the human skull and jawbone
were extremely old

SKILL 4: FIND DIRECT ANSWERS ABOUT PART OF THE PASSAGE


Many questions in the Reading section of the TOEFL ask about one small part of the
passage rather than the passage as a whole. This means that you can find the answers
directly in the passage. The following are examples of this type of question:

QUESTIONS ABOUT WHAT IS IN THE PASSAGE


 ACCORDING TO THE PASSAGE ...
 IT IS STATED IN THE PASSAGE THAT ...
This type of question asks for information that is explicitly given in the passage. The
answers to these questions are generally given in order in the passage, and the correct
answer is often a restatement of what is given in the passage. This means that the

125
TOEFL Preparation Exercises Reading Comprehension & Vocabulary
correct answer often expresses the same idea as what is written in the passage, but the
words are not exactly the same. For example, if the passage states that George
Washington's family was rich, then the correct answer to a question about the
financial status of Washington's family might state that his relatives were wealthy.

QUESTIONS ABOUT WHAT IS NOT IN THE PASSAGE

 WHICH OF THE FOLLOWING IS NOT TRUE?


 WHICH OF THE FOLLOWING IS NOT STATED IN THE PASSAGE?
 ALL OF THE FOLLOWING ARE TRUE EXCEPT ...
This type of question asks about what is not in the passage or what is not true
according to the passage. In this type of question, three of the answers are true and
one of the answers is not mentioned in the passage or is not true according to the
passage. The correct answer is the answer that is not true or that is not mentioned in
the passage. Many students, however, will make a mistake and choose one of the true
answers.

STRATEGIES FOR ANSWERING DIRECT QUESTIONS ABOUT PART OF THE


PASSAGE

When answering this type of question, you should remember that (1) these detail
questions are generally answered in order in the passage, and (2) the correct answers
can use exactly the same words as the passage but generally are restatements of what
is said. The following method works best for questions about what is or is not in the
passage.

STRATEGIES: DIRECT QUESTIONS ABOUT PART OF THE PASSAGE

1. Decide where to look in the passage for the correct answer, based on your
understanding of the organization of details in the passage and the
knowledge that questions are generally answered in order in the passage.
2. Eliminate any definitely wrong answers and choose the best of the remaining
ones.

3. If the question says NOT or EXCEPT, choose the answer that is not true or
riot stated in the passage. Answers that are true according to the passage are

126
TOEFL Preparation Exercises Reading Comprehension & Vocabulary
not correct in this type of question.

Example

Williamsburg is a historic city in Virginia situated on a peninsula between


two rivers, the York and the James. It was settled by English colonists in 1633,
twenty-six years after the first permanent English colony in America was settled
at Jamestown. In the beginning the colony at Williamsburg was named Middle
Plantation because of its location in the middle of the peninsula. The site for
Williamsburg had been selected by the colonists because the soil drainage was
better there than at the Jamestown location, and there were fewer mosquitoes.

1. According to the passage, Williamsburg is located


(A) on an island
(B) in the middle of a river
(C) where the York and the James meet
(D) on a piece of land with rivers on two sides

The answers to questions are generally found in order in the passage, so you should
look for the answer to the first question near the beginning of the passage. Since the
question asks about where Williamsburg is located, you should see that the first
sentence in the passage answers the question because situated means located.
Answer (A) is an incorrect answer because Williamsburg is not located on an island;
the passage states that it is situated on a peninsula. Answer (B) is incorrect because
Williamsburg is between two rivers, not in the middle of a river. Answer (C) is
incorrect because the passage says nothing about whether or not the two rivers meet
at Williamsburg. The best answer to this question is answer (D); rivers on two sides
is closest in meaning to between two rivers.

2. The passage states that the name Middle Plantation


(A) is a more recent name than Williamsburg
(B) derived from the location of the colony on the peninsula
(C) refers to the middle part of England that was home to the colonists
(D) was given to the new colony because it was located in the middle
of several plantations

The answer to the second question will probably be located in the passage after the
answer to the first question. Because the question is about the name Middle
Plantation, you should skim through the passage to find the part that discusses this

127
TOEFL Preparation Exercises Reading Comprehension & Vocabulary
topic. The answer to this passage is found in the statement Williamsburg was named
Middle Plantation because of
its location in the middle in
of the peninsula. Answer (B) is correct because it is closest
meaning to this statement. Answer (A) is incorrect because it is false; the area was
named Middle Plantation in the beginning, and the name Williamsburg is more
recent. Answer (C) is incorrect because the passage says nothing about naming the
area after the colonists' home in England. Answer (D) is incorrect because the
passage says nothing about any other plantations in the area of Williamsburg.

3. Which of the following is NOT true, according to the passage?


(A) Jamestown was settled before Williamsburg.
(B) Williamsburg was settled by English colonists.
(C) Soil drainage was not as good at Jamestown as at Williamsburg.
(D) There were more mosquitoes at Williamsburg than at Jamestown.
In a question about what is not true, three of the answers are true and only one is
not true; the correct answer is the one that is not true. Answer (A) is true because the
passage states that Williamsburg was settled ... twenty-six years after the ... colony
... at Jamestown. Since answer (A) is true, it is not correct. Answer (B) is also true
because the passage clearly states that Williamsburg was settled by English
colonists. Since answer (B) is true, it is not correct. Answer (C) is true because the
passage states that soil drainage was better at Williamsburg than at the Jamestown
location. Since answer (C) is true, it is not correct. The best answer to this question is
answer (D) because it is not true: according to the passage there were fewer
mosquitoes at Williamsburg than at Jamestown.

EXERCISE 4: Study each passage and chose the best answers to the questions that
follow.

PASSAGE ONE (Questions 1-3)

Ice ages, those periods when ice covered extensive areas of the earth, are
known to have occurred at least six times. Past ice ages can be recognized from
rock strata that show evidence of foreign materials deposited by moving walls of
ice or melting glaciers. Ice ages can also be recognized from land formations that
have been produced from moving walls of ice, such as U-shaped valleys,
sculptured landscapes, and polished rock faces.

1. According to the passage, what happens during an ice age?


(A) Rock strata are recognized by geologists.
(B) Evidence of foreign materials is found.

128
TOEFL Preparation Exercises Reading Comprehension & Vocabulary
(C) Ice covers a large portion of the earth's surface.
(D) Ice melts six times.
2. The passage covers how many different methods of recognizing past ice ages?
(A) One
(B) Two
(C) Three
(D) Four
3. According to the passage, what in the rock strata is a clue to geologists of a past ice
age?
(A) Ice
(B) Melting glaciers
(C) U-shaped valleys
(D) Substances from other areas

PASSAGE TWO (Questions 4-6)

The human heart is divided into four chambers, each of which serves its own
function in the cycle of pumping blood. The atria are the thin-walled upper cham-
bers that gather blood as it flows from the veins between heartbeats. The ven-
tricles are the thick-walled lower chambers that receive blood from the atria and
push it into the arteries with each contraction of the heart. The left atrium and
ventricle work separately from those on the right. The role of the chambers on the
right side of the heart is to receive oxygen-depleted blood from the body tissues
and send it on to the lungs; the chambers on the left side of the heart then receive
the oxygen-enriched blood from the lungs and send it back out to the body
tissues.
4. All of the following are true about ventricles EXCEPT that
(A) they have relatively thick walls.
(B) they send blood to the atria.
(C) they are lower than the atria.
(D) there are two ventricles, a right one and a left one.
5. When is blood pushed into the arteries from the ventricles?
(A) As the heart beats
(B) Between heartbeats
(C) Before each contraction of the heart
(D) Before it is received by the atria

129
TOEFL Preparation Exercises Reading Comprehension & Vocabulary
6. According to the passage, which part
of the heart gets blood from the body
tissues and passes it on to the lungs?
(A) The atria
(B) The ventricles
(C) The right atrium and ventricle
(D) The left atrium and ventricle
PASSAGE THREE (Questions 7-9)
The Golden Age of railroads refers to the period from the end of the Civil War
to the beginning of World War I when railroads flourished and in fact maintained
a near monopoly in mass transportation in the United States. One of the
significant developments during the period was the notable increase in uniform-
ity, particularly through the standardization of track gauge and time.
At the end of the Civil War, only about half of the nation's railroad track was
laid at what is now the standard gauge of 1.4 meters; much of the rest,
particularly in southern states, had a 1.5-meter gauge. During the post-war years
tracks were converted to the 1.4-meter gauge, and by June 1, 1886, the stan-
dardization of tracks was completed, resulting in increased efficiency and econ-
omy in the rail system.
A further boon to railroad efficiency was the implementation of Standard
Time in 1883. With the adoption of Standard Time, four time zones were estab-
lished across the country, thus simplifying railroad scheduling and improving the
efficiency of railroad service.

7. Which of the following is NOT true about the Golden Age of railroads, according
to the passage?
(A) It occurred prior to the first World War.
(B) Most of U.S. mass transportation was controlled by the railroads.
(C) Track gauge was standardized before the Golden Age of railroads.
(D) Standard Time was implemented during the Golden Age of railroads.
8. According to the passage, the establishment of uniformity of track gauge resulted
in which of the following?
(A) The Civil War
(B) Improved economy in the transportation system
(C) Standardization of time zones
(D) Railroad schedules

130
TOEFL Preparation Exercises Reading Comprehension & Vocabulary
9. According to the passage, when was Standard Time implemented in the United
States?
(A) Before the Civil War
(B) On June 1, 1886
(C) After World War I
(D) Before standardized track gauge was established throughout the U.S.

REVIEW EXERCISE (SKILLS 1-4): Study each of the passages and choose the
best answers to the questions that follow.

PASSAGE ONE (Questions 1-4)

Lincoln's now famous Gettysburg Address was not, on the occasion of its
delivery, recognized as the masterpiece it is today. Lincoln was not even the
primary speaker at the ceremonies, held at the height of the Civil War in 1863, to
dedicate the battlefield at Gettysburg. The main speaker was orator Edward
Everett, whose two-hour speech was followed by Lincoln's shorter remarks. Lin-
coln began his small portion of the program with words that today are immediately
recognized by most Americans: "Four score and seven years ago our fathers
brought forth on this continent a new nation, conceived in liberty and dedicated to
the proposition that all men are created equal." At the time of the speech, little
notice was given to what Lincoln had said, and Lincoln considered his appearance
at the ceremonies rather unsuccessful. It was after his speech appeared in print
that it began receiving the growing recognition that today places it among the
greatest speeches of all time.

1. The main idea of this passage is that


(A) the Gettysburg Address has always been regarded as a masterpiece
(B) at the time of its delivery the Gettysburg Address was truly appreciated as
a masterpiece
(C) it was not until sometime after 1863 that Lincoln's speech at Gettysburg took
its place in history
(D) Lincoln is better recognized today than he was at the time of his presidency
2. Which of the following is true about the ceremonies at Gettysburg during the Civil
War?
(A) Lincoln was the main speaker.
(B) Lincoln gave a two-hour speech.
(C) Lincoln was the opening speaker of the ceremonies.

131
TOEFL Preparation Exercises Reading Comprehension & Vocabulary
(D) Lincoln's first public words were: "Four score and seven years ago ......
3. According to the passage, when Lincoln spoke at the Gettysburg ceremonies,
(A) his words were immediately recognized by most Americans
(B) he spoke for only a short period of time
(C) he was enthusiastically cheered
(D) he was extremely proud of his performance
4. When did Lincoln's Gettysburg Address begin to receive public acclaim?
(A) After it had been published
(B) Immediately after the speech .
(C) Not until the present day
(D) After Lincoln received growing recognition

PASSAGE TWO (Questions 5-8)


Hay fever is a seasonal allergy to pollens; the term hay fever, however, is a less
than adequate description since an attack of this allergy does not incur fever and
since such an attack can be brought on by sources other than hay-producing
grasses. Hay fever is generally caused by air-borne pollens, particularly ragweed
pollen. The amount of pollen in the air is largely dependent on geographical lo-
cation, weather, and season. In the eastern section of the United States, for
example, there are generally three periods when pollen from various sources can
cause intense hay fever suffering: in the springtime months of March and April
when pollen from trees is prevalent, in the summer months of June and July when
grass pollen fills the air, and at the end of August when ragweed pollen is at its
most concentrated levels
5. Which of the following would be the best title for the passage?
(A) The Relationship Between Season and Allergies
(B) Misconceptions and Facts about Hay Fever
(C) Hay Fever in the Eastern U.S.
(D) How Ragweed Causes Hay Fever
6. According to the passage, which of the following helps to explain why the term hay
fever is somewhat of a misnomer?
(A) A strong fever occurs after an attack.
(B) The amount of pollen in the air depends on geographical location.
(C) Hay fever is often caused by ragweed.
(D) Grass pollen is prevalent in June and July.
7. Which of the following is NOT discussed in the passage as a determining factor of
the amount of pollen in the air?

132
TOEFL Preparation Exercises Reading Comprehension & Vocabulary
(A) Place
(B) Climate
(C) Time of year
(D) Altitude
8. Which of the following is NOT true about hay fever in the eastern U.S.?
(A) Suffering from hay fever is equally severe year-round.
(B) Pollen from trees causes hay fever suffering in the spring.
(C) Grass pollen fills the air earlier in the year than ragweed pollen.
(D) Ragweed pollen is most prevalent at the end of the summer.

PASSAGE THREE (Questions 9-12)


According to the theory of continental drift, the continents are not fixed in
position but instead move slowly across the surface of the earth, constantly
changing in position relative to one another. This theory was first proposed in
the eighteenth century when mapmakers noticed how closely the continents of
the earth fit together when matched up. It was suggested then that the present-
day continents had once been one large continent that had broken up into pieces
which drifted apart.
Today the modern theory of plate tectonics has developed from the theory of
continental drift. The theory of plate tectonics suggests that the crust of the
earth is divided into six large, and many small, tectonic plates that drift on the
lava that composes the inner core of the earth. These plates consist of ocean
floor and continents that quite probably began breaking up and moving relative
to one another more than 200 million years ago.

9. The topic of this passage is


(A) continental drift
(B) the theory of plate tectonics
(C) the development of ideas about the movement of the earth's surface
(D) eighteenth-century mapmakers
10. The passage states that the theory of continental drift developed as a result of
(A) the fixed positions of the continents
(B) the work of mapmakers
(C) the rapid movement of continents
(D) the fit of the earth's plates

133
TOEFL Preparation Exercises Reading Comprehension & Vocabulary
11. Which of the following is NOT true about the theory of plate tectonics?
(A) It is not as old as the theory of continental drift.
(B) It evolved from the theory of continental drift.
(C) It postulates that the earth's surface is separated into plates.
(D) It was proposed by mapmakers.
12. According to the passage, what constitutes a tectonic plate?
(A) The inner core of the earth
(B) Only the continents
(C) Lava
(D) The surface of the land and the floor of the oceans

SKILL 5: FIND INDIRECT ANSWERS ABOUT PART OF THE PASSAGE


Some questions will ask for answers that are not given directly in the passage. To
answer these questions correctly, you will have to draw conclusions from
information that is given in the passage. These questions can be about the whole
passage (as in Skill 6), or they can be about only one small part of the passage.
The following are examples of indirectly answered questions about only part of
the passage:

QUESTIONS ABOUT WHAT IS IMPLIED IN THE PASSAGE


 IT IS IMPLIED IN THE PASSAGE THAT ...
 THE AUTHOR INFERS THAT .. .
 IT IS MOST LIKELY THAT ...
 WHAT PROBABLY HAPPENED ...

The words implied, inferred, most likely, or probably are clues to you that the
question is not answered directly in the passage. To answer such a question, you
will have to draw a conclusion from something in the passage. For example, if the
passage states that George Washington attended private schools, owned five
horses, and enjoyed wearing the latest fashions, you could draw the conclusion
that he was rich even though the passage does not say so directly.
Keep in mind when you are looking for some information to help you answer this
type of question that the questions in the Reading section of the TOEFL are
generally answered in order in the passage. For example, if the answer to
question 35 is implied in the passage, the information that will help you answer
this question will come in the passage after the answer to question 34 and before
the answer to question 36.

134
TOEFL Preparation Exercises Reading Comprehension & Vocabulary
QUESTIONS ABOUT THE MEANINGS OF VOCABULARY WORDS
 THE WORD "INEVITABLE" IN LINE 6 MEANS ...
A question about the meaning of a particular vocabulary word is generally not
answered directly in the Reading section of the TOEFL. For this type of question
you must draw a conclusion about the meaning of the word from information in
the passage. For example, the passage might state that George Washington was
very disheartened during the long winter at Valley Forge because of the
tremendous losses his troops had suffered, and the question might ask the
meaning of the word disheartened. To answer this question, you should study the
context around the word that is being tested. The sentence indicates that
Washington's troops had suffered tremendous losses, so Washington probably
felt discouraged. The word disheartened means discouraged.

QUESTIONS ABOUT WHAT COMES BEFORE OR AFTER THE PASSAGE


 WHAT INFORMATION IS PROBABLY IN THE PARAGRAPH
PRECEDING THE PASSAGE?
 THE PARAGRAPH FOLLOWING THE PASSAGE MOST LIKELY
CONTAINS WHAT INFORMATION?
A question about what probably came before the passage or what most likely
comes next in the passage is not answered directly in the passage. You can draw a
conclusion from the information in the passage to answer this type of question.
To answer a question about what probably came before this passage, you
should look at the beginning of the passage: the first sentence of the passage is
probably a transition from the previous paragraph and will tell you what probably
came before. For example, perhaps a passage begins with after serving as the
Commander-in-Chief of the Revolutionary forces, 'George Washington was
elected as the first President of the United States. This probably means that the
previous paragraph was about how George Washington served as Commander-
in-Chief of the Revolutionary forces and the passage that follows is about his
election as the first U.S. president. I
"To answer a question about what probably comes after the passage, you
should look at the end of the passage: the last sentence is probably a transition
from this passage to the next paragraph. For example, a passage might end with
after his election to the presidency, George Washington began looking for new
ways to strengthen the young nation. The next paragraph is probably about
George Washington's new ways to strengthen the young nation.

135
TOEFL Preparation Exercises Reading Comprehension & Vocabulary

STRATEGIES FOR ANSWERING INDIRECT QUESTIONS ABOUT PART OF THE


PASSAGE

For this type of question you should remember that (1) implied, inferred, most
likely, and probably are clues that the question is not answered directly in the
passage, and (2) the questions are generally answered in order in the passage.

STRATEGIES:
INDIRECT QUESTIONS ABOUT PART OF THE PASSAGE

1. Decide where to look in the passage for the correct answer based on your un-
derstanding of the organization of details in the passage and the knowledge that
questions are generally answered in order in the passage.
2. Draw a conclusion from the information that is given to answer the question.
Do not expect to find a direct answer to the question.

Example

Another program instrumental in the popularization of science was Cosmos.


This series, broadcast on public television, dealt with topics and issues from var-
ied fields of science. The principal writer and narrator of the program was Carl
Sagan, a noted astronomer and Pulitzer Prize winning author.

1. The paragraph preceding this passage most probably discusses


(A) a different scientific television series
(B) Carl Sagan's scientific achievements
(C) the Pulitzer Prize won by Carl Sagan
(D) public television

A question about what probably came before the passage is generally answered in
the beginning of the passage; you should therefore carefully study the first line of
this passage to answer the question. Since this passage about the scientific television
program Cosmos begins with another program, it should be clear that the paragraph
preceding this passage probably discusses another scientific television program.
Therefore the best answer to this question is answer (A).

136
TOEFL Preparation Exercises Reading Comprehension & Vocabulary
2. The word "issues" (line 2) is closest in meaning to which of the following words?
(A) Editions
(B) Volumes
(C) Solutions
(D) Matters of public concern
A question about the meaning of a vocabulary word is generally not answered
directly in the passage; you must instead determine as much as you can about the
meaning of the word from the context. Since the passage states that the television
program discusses topics and issues, hopefully you can understand that the best
answer to this question is answer (D) matters of public concern. Be very careful to
read the context in this type of question. Issues can mean editions (answer A) or
volumes (answer B) when it refers to publications. However, in this context, issues
do not mean editions or volumes. Answer (C) is incorrect because issues are more
related to problems than to solutions.
3. Which of the following would probably NOT be the title of a Cosmos show?
(A) Is There Life on Mars?
(B) Exploring the Human Brain
(C) How to Deal with World Poverty
(D) When Will the Next Big Earthquake Occur?

Since the question says probably, the answer is not given directly in the passage;
however, you can draw a conclusion from something in the passage to answer the
question. The passage states that Cosmos was about topics and issues from varied
fields of science. Answer (A) about Mars, answer (B) about the human brain, and
answer (D) about earthquakes are science topics and could therefore possibly serve
as topics of Cosmos shows. Since World Poverty (answer C) is more a question of
sociology or economics than science, the best answer to this question is answer (C).

EXERCISE 5: Study each of the passages and choose the best answers to the questions
that follow.

PASSAGE ONE (Questions 1-4)

The most conservative sect of the Mennonite Church is the Old Order
Amish, with 33,000 members living mainly today in the states of Pennsylvania,
Ohio, and Indiana. Their lifestyle reflects their belief in the doctrines of
separation from the world and simplicity of life. The Amish have steadfastly
rejected the societal changes that have occurred in the previous three hundred
years, preferring instead to remain securely rooted in a seventeenth-century
lifestyle. They live without radios, televisions, telephones, electric lights, and

137
TOEFL Preparation Exercises Reading Comprehension & Vocabulary
cars; they dress in plainly styled and colored old-fashioned clothes; and they
farm their lands with horses and tools rather than modern farm equipment.
They have a highly communal form of living, with barn raisings and quilting
bees as commonplace activities.

1. The paragraph preceding this passage most probably discusses


(A) other more liberal sects of Mennonites
(B) where Mennonites live
(C) the communal Amish lifestyle
(D) the most conservative Mennonites
2. The word "steadfastly" (line 4) is closest in meaning to
(A) quickly
(B) staunchly
(C) negatively
(D) simply
3. Which of the following would probably NOT be found on an Amish farm?
(A) A hammer
(B) A cart
(C) A long dress
(D) A refrigerator
4. It can be inferred from the passage that a quilting bee
(A) involves a group of people
(B) is necessary when raising bees
(C) always follows a barn raising
(D) provides needed solitude
PASSAGE TWO (Questions 5-8)
Various other Indian tribes also lived on the Great Plains. The Sioux, a
group of seven American Indian tribes, are best known for their fiercely
combative posture against the encroaching white civilization in the 1800's.
Although they are popularly referred to today as Sioux, these Indian tribes
did not call themselves Sioux; the name of Sioux was given to them by an
enemy tribe. The seven Sioux tribes called themselves by some variation of
the word Dakota, which means "allies" in their language. Four tribes of the
eastern Sioux community living in' Minnesota were known by the name
Dakota. The Nakota included two tribes that left the eastern woodlands and
moved out onto the plains. The Teton Sioux, or Lakota, moved even further
west to the plains of the present-day states of North Dakota, South Dakota, and
Wyoming.

138
TOEFL Preparation Exercises Reading Comprehension & Vocabulary
5. The paragraph preceding this passage most probably discusses
(A) how the Sioux battled the white man
(B) one of the Plains Indian tribes
(C) where the Sioux lived
(D) American Indian tribes on the east coast
6. The word "encroaching" (line 3) is closest in meaning to which of the
following?
(A) Attacking
(B) Dominant
(C) Intruding
(D) Withdrawing
7. It is implied in the passage that the seven Sioux tribes called each other by
some form of the word "Dakota" because they were
(A) united in a cause
(B) all living in North Dakota
(C) fiercely combative
(D) enemies
8. It can be inferred from the passage that the present-day states of North
and South Dakota
(A) are east of Minnesota
(B) are home to the four tribes known by the name "Dakota"
(C) received their names from the Indian tribes living there
(D) are part of the eastern woodlands,

PASSAGE THREE (Questions 9-12)


The extinction of many species of birds has undoubtedly been hastened by
modern man; since 1600 it has been estimated that approximately 100 bird species
have become extinct over the world. In North America, the first species known to be
annihilated was the great auk, a flightless bird that served as an easy source of food
and bait for Atlantic fishermen through the beginning of the nineteenth century.
Shortly after the great auk's extinction, two other North American species, the
Carolina parakeet and the passenger pigeon, began dwindling noticeably in
numbers. The last Carolina parakeet and the last passenger pigeon in captivity both
died in September 1914. In addition to these extinct species, several others such as
the bald eagle, the peregrine falcon, and -the California condor are today recognized
as endangered steps are being taken to prevent their extinction.

139
TOEFL Preparation Exercises Reading Comprehension & Vocabulary
9. The passage implies that the great 11. The word "annihilated" (line 4)
is auk disappeared closest in meaning to which of the
(A) before 1600 following?
(B) in the 1600s (A) Wiped out
(C) in the 1800s (B) Decreased
(D) in the last fifty years (C) Hurt
10. It can be inferred from the passage (D) Damaged
that the great auk was killed beause 12. The paragraph following this
passage most probably discusses
(A) it was eating the fishermen's (A) what is being done to save
catch endangered birds
(B) fishermen wanted to eat it (B) what the bald eagle
symbolizes to Americans
(C) it flew over fishing areas
(D) it baited fishermen (C) how several bird species
became endangered
(D) other extinct species

REVIEW EXERCISE (SKILLS 1-5): Study each of the passages and choose the
best answers to the questions that follow.

PASSAGE ONE (Questions 1-5)


The Mason-Dixon Line, often considered by Americans to be the demarcation
between the North and the South, is in reality the boundary that separates the state of
Pennsylvania from Maryland and parts of West Virginia. Prior to the Civil War, this
southern boundary of Pennsylvania separated the non-slave states to the north from
the slave states to the south.
The Mason-Dixon Line was established well before the Civil War, as a result of a
boundary dispute between Pennsylvania and Maryland. Two English astronomers,
Charles Mason and Jeremiah Dixon, were called in to survey the area and officially
mark the boundary between the two states. The survey was completed in 1767, and the
boundary was marked with stones.

1. The best title for this passage would be


(A) Dividing the North and the South
(B) The Meaning of the Mason-Dixon Line
(C) Two English Astronomers
(D) The History of the Mason-Dixon Line

140
TOEFL Preparation Exercises Reading Comprehension & Vocabulary
2. It can be inferred from the passage that before the Civil War
(A) Pennsylvania was south of the Mason-Dixon Line
(B) Pennsylvania was a non-slave state
(C) the states south of the Mason-Dixon Line had the same opinion about slavery
as Pennsylvania
(D) the slave states were not divided from the non-slave states
3. According to the passage, the Mason-Dixon Line was established because of a
disagreement
(A) about borders
(B) about slaves
(C) between two astronomers
(D) over surveying techniques
4. The word "survey" (line 8) is closest in meaning to which of the following?
(A) View
(B) Understand
(C) Map out
(D) Separate
5. The passage indicates that the Mason-Dixon Line was identified with
(A) pieces of rock
(B) a fence
(C) a stone wall
(D) a border crossing

PASSAGE TWO (Questions 6-10)

Manic depression is another psychiatric illness that mainly affects the mood. A
patient suffering from this disease will alternate between periods of manic
excitement and extreme depression, with or without relatively normal periods in
between. The changes in mood suffered by a manic-depressive patient go far beyond
the day-to-day mood changes experienced by the general population. In the period of
manic excitement, the mood elevation can become so intense that it can result in
extended insomnia, extreme irritability, and heightened aggressiveness. In the period
of depression, which may last for several weeks or months, a patient experiences
feelings of general fatigue, uselessness, and hopelessness, and in serious cases may
contemplate suicide.

141
TOEFL Preparation Exercises Reading Comprehension & Vocabulary

6. The paragraph preceding this passage most probably discusses


(A) when manic depression develops
(B) a different type of mental disease
(C) how moods are determined
(D) how manic depression can result in suicide
7. According to the passage, a manic-depressive patient in a manic phase would be
feeling
(A) highly emotional
(B) unhappy
(C) listless
(D) relatively normal
8. The passage indicates that most people
(A) never undergo mood changes
(B) experience occasional shifts in mood
(C) switch wildly from highs to lows
(D) become highly depressed
9. The word "intense" (line 6) is closest in meaning to
(A) lengthy
(B) haughty
(C) strong
(D) extensive
10. The passage implies that
(A) changes from excitement to depression occur frequently and often
(B) only manic-depressive patients experience aggression
(C) the depressive phase of this disease can be more harmful than the manic
phase
(D) suicide is inevitable in cases of manic depression

PASSAGE THREE (Questions 11-15)


The 1960 presidential campaign featured the politically innovative and highly
influential series of televised debates in the contest between the Republicans and
the Democrats. Senator John Kennedy established an early lead among the Dem-
ocratic hopefuls and was nominated on the first ballot at the Los Angeles con-
vention to be the representative of the Democratic party in the presidential election.
Richard Nixon, then serving as Vice-President of the United States under
Eisenhower, received the nomination of the Republican party. Both Nixon and

142
TOEFL Preparation Exercises Reading Comprehension & Vocabulary
Kennedy campaigned vigorously throughout the country and then took the un-
precedented step of appearing in face-to-face debates on television. Political experts
contend that the debates were a pivotal force in the elections. In front of a
viewership of more 'than 100 million citizens, Kennedy masterfully overcame
Nixon's advantage as the better-known and more experienced candidate and re-
versed the' public perception of him as too inexperienced and immature for the
presidency.

11. Which of the following best expresses the main idea of the passage?
(A) Kennedy defeated Nixon in the 1960 presidential election.
(B) Television debates were instrumental in determining the outcome of the
1960 presidential election.
(C) Television debates have long been a part of campaigning.
(D) Kennedy was the leading Democratic candidate in the 1960 presidential
election.
12. The passage implies that Kennedy
(A) was a long shot to receive the Democratic presidential nomination
(B) won the Democratic presidential nomination fairly easily
(C) was not a front runner in the race for the Democratic presidential
nomination
(D) came from behind to win the Democratic presidential nomination
13. The passage states that the television debates between presidential candidates in
1960
(A) did not establish a precedent
(B) were the final televised presidential debates
(C) were fairly usual in the history of presidential campaigns
(D) were the first presidential campaign debates to be televised
14. The word "pivotal" (line 10) is closest in meaning to
(A) circular
(B) influential
(C) nebulous
(D) uneven
15. The passage states that in the debates with Nixon, Kennedy demonstrated to the
American people that he was
(A) old enough to be president
(B) more experienced than Nixon
(C) better known than Nixon
(D) too inexperienced to serve as president

143
TOEFL Preparation Exercises Reading Comprehension & Vocabulary

SKILL 6: FIND INDIRECT ANSWERS ABOUT THE WHOLE PASSAGE


Some questions about the whole passage will ask for answers that are not given
directly in the passage. To answer these questions correctly, you must draw
conclusions from information in the passage. The following are examples of
questions about the whole passage that are not answered directly for you in the
passage:
QUESTIONS ABOUT TONE
 WHAT IS THE TONE OF THE PASSAGE?
A question about the tone is asking if the author is showing any emotion in his or
her writing. The majority of the passages on the TOEFL are factual passages
presented without any emotion; the tone of this type of passage could be simply
informational, explanatory, or factual. Sometimes on the TOEFL, however,
the author shows some emotion in his or her writing, and you must be able to
recognize that emotion to answer a question about tone correctly. If the author is
being funny, then the tone might be humorous; if the author is making fun of
something, the tone might be sarcastic; if the author feels strongly that
something is wrong or right, the tone might be impassioned.
QUESTIONS ABOUT PURPOSE
 WHAT IS THE AUTHOR'S PURPOSE IN THIS PASSAGE?
A question about purpose is asking what the author is trying to do in the passage.
You can draw a conclusion about the author's purpose by referring to the main
idea and the organization of details in the passage. For example, if the main idea is
that George Washington's early life greatly influenced his later career and if the
details give a history of his early life, the author's purpose could be to show how
George Washington's early life influenced his later career. However, the
answer to a purpose question is often considerably more general than the main
idea. A more general author's purpose for the main idea about George Washington
would be to demonstrate the influence of early experiences on later life (without
any mention of George Washington).
QUESTIONS ABOUT THE COURSE
 IN WHICH COURSE WOULD THIS READING PROBABLY BE
ASSIGNED?
A question about the course is asking you to decide which university course might
have this passage as assigned reading. You should draw a conclusion about the
course by referring to the topic of the passage and the organization of details. For
example, if the passage is about George Washington and the details give historical

144
TOEFL Preparation Exercises Reading Comprehension & Vocabulary
background on his early life, then this would probably be assigned reading in an
American history class. However, if the passage is about George Washington and
the details show the various influences that George had on the formation of the
American government, then the passage might be assigned reading in a
government or political science class.
STRATEGIES FOR ANSWERING INDIRECT QUESTIONS ABOUT THE WHOLE
PASSAGE
Questions about tone, purpose, or course are all asking you to draw a conclusion about
the whole passage. When you are answering this type of question, you should
remember that (1) the answer is not given directly, and (2) the answer must refer to the
passage as a whole rather than one detail in the passage. The following strategies work
best for questions about tone, purpose, or course.

STRATEGIES:
INDIRECT QUESTIONS ABOUT THE WHOLE PASSAGE

1. Tone: As you read the passage, look for clues that the author is showing some
emotion rather than just presenting facts.
2. Purpose: Draw a conclusion about the purpose from the main idea of the
passage and the overall organization of details.

3. Course: Draw a conclusion about the course from the topic of the passage
and the organization of details.

Example

Military awards have long been considered symbolic of royalty, and thus when
the United States was a young nation just finished with revolution and eager to
distance itself from anything tasting of monarchy, there was strong sentiment
against military decoration. For a century, from the end of the Revolutionary War
until the Civil War, the United States awarded no military honors. The institution
of the Medal of Honor in 1861 was a source of great discussion and concern. From
the Civil War until World War I, the Medal of Honor was the only military award
given by the United States government, and today it is awarded only in the most
extreme cases of heroism. Although the United States is still somewhat wary of
granting military awards, several awards have been instituted since World War I.

145
TOEFL Preparation Exercises Reading Comprehension & Vocabulary
1. The tone of the passage is
(A) angered
(B) humorous
(C) outraged
(D) informational
To determine the tone of a passage, you should look for any indications of emotion on
the part of the author. In this passage the author uses historical facts to make a point
about America's sentiment against military awards; the author does not make any kind
of emotional plea. Therefore, the best answer to this question is answer (D). There is
nothing in this passage to indicate any anger (A), or humor (B), or outrage (C) on the
part of the author.
2.The author's purpose in this passage is to
(A) describe the history of military awards from the Revolutionary War to the Civil War
(B) demonstrate an effect of America's attitude toward royalty
(C) give an opinion of military awards
(D) outline various historical symbols of royalty
To answer this question correctly, refer to the main idea of this passage as
outlined in the first sentence. The main idea is that there has been strong sentiment
against military awards in the United States because military awards are symbols of
royalty. The author gives historical facts about military awards as details to support the
main idea. Since the purpose is determined from the main idea and the overall
organization of details, the author's purpose is to describe, explain, or demonstrate that
America's sentiment against military awards is because of its negative sentiment
against royalty.
The best answer to this question is therefore answer (B) because this answer is
closest in meaning to the purpose that you have determined from the main idea and
organization of details. Notice that the correct answer is considerably more general
than the main idea: according to answer (B), the purpose is to demonstrate an effect
(America's dislike of military awards) of America's attitude toward royalty. Answer (A)
is incorrect because it refers to only one part of the passage, military awards from the
Revolutionary War to the Civil War, while the passage discusses military awards from
the Revolutionary War to the present. Answer (C) is incorrect because the author is not
giving his or her own opinion of military awards in this passage, but is instead
attempting to describe the sentiment of the American people. Answer (D) is incorrect
because the author does not attempt to describe the various symbols of royalty in this
passage. Only one symbol of royalty (military awards) is discussed, but there are many
other royal symbols.
3.This passage would probably be assigned reading in a course on
(A) military science
(B) sociology

146
TOEFL Preparation Exercises Reading Comprehension & Vocabulary
(C) American history
(D) interior decoration
To draw a conclusion about the course, you should refer to the topic of the
passage and the overall organization of details. Since this passage is about the
American military awards, and the details discuss the history of American
military awards from the Revolutionary War until today, the best answer is (C).
EXERCISE 6: Study each of the passages and choose the best answers to the questions
that follow.

PASSAGE ONE (Questions 1-3)

In Cold Blood (1966) is a well-known example of the "nonfiction novel," a


recently popular type of writing based upon factual events in which the author
attempts to describe the underlying forces, thoughts, and emotions that lead to
actual events. In Truman Capote's book, the author describes the sadistic murder
of a family on a Kansas farm, often showing the point of view of the killers. To
research the book, Capote interviewed the murderers, and he maintains that his
book presents a faithful reconstruction of the incident.

3. This passage would probably be assigned


1. The purpose of this passage is to
reading in which of the following
(A) discuss an example of a courses?
new literary genre (A) Criminal Law
(B) tell the story of In Cold Blood (B) American History
(C) explain Truman Capote's (C) Modern American Novels
reasons
for writing In Cold Blood (D) Literary Research
(1)) describe how Truman
Capote
researched his nonfiction novel
2. Which of the following best
describes the tone of the passage?
(A) Cold
(B) Sadistic
(C) Emotional
.(D) Descriptive

147
TOEFL Preparation Exercises Reading Comprehension & Vocabulary
PASSAGE TWO (Questions 4-6)
Up to now confessions that have been obtained from defendants in a hypnotic
state have not been admitted into evidence by courts in the United States. Experts in
the field of hypnosis have found that such confessions are not completely reliable.
Subjects in a hypnotic state may confess to crimes they did not commit for one of two
reasons. Either they fantasize that they had committed the crimes or they believe that
others want them to confess.
A landmark case concerning a confession obtained under hypnosis went all the way
to the U.S. Supreme Court. In the case of Leyra vs. Denno, a suspect was hypnotized by
a psychiatrist for the district attorney; in a post-hypnotic state the suspect signed three
separate confessions to a murder. The Supreme Court ruled that the confessions were
invalid because of the mental coercion used to obtain them. The suspect was then
released because the confessions had been the only evidence against him.

4. Which of the following best describes the author's purpose in this passage?
(A) To describe the history of hypnosis
(B) To demonstrate why confessions made under hypnosis are not reliable
(C) To clarify the role of the Supreme Court in invalidating confessions from
hypnotized subjects
(D) To explain the legal status of hypnotically induced confessions

5. The tone of this passage could best be described as


(A) outraged
(B) judicious
(C) hypnotic
(D) informative
6. This passage would probably be assigned reading in a course on
(A) American Law
(B) Psychiatric Healing
(C) Parapsychology
(D) Philosophy

PASSAGE THREE (Questions 7-9)


The rate at which the deforestation of the world is proceeding is alarming. In
1950 approximately 25 percent of the earth's land surface had been covered with
forests, and less than 25 years later the amount of forest land was reduced to 20
percent. This decrease from 25 percent to 20 percent from 1950 to 1973 represents
an astounding loss of 20 million square kilometers of forests. Predictions are that

148
TOEFL Preparation Exercises Reading Comprehension & Vocabulary
an additional 20 million square kilometers of forest land will be lost by 2020.
The majority of deforestation is occurring in tropical forests in developing
countries, fueled by the developing countries' need for increased agricultural land
and the desire on the part of developed countries to import wood and wood
products. More than 90 percent of the plywood used in the United States, for
example, is imported from developing countries with tropical forests. By the mid-
1980's, solutions to this expanding problem were being sought, in the form of
attempts to establish an international regulatory organization to oversee the use of
tropical forests.
7. The author's main purpose in this passage is to
(A) cite statistics about an improvement on the earth's land surface
(B) explain where deforestation is occurring
(C) make the reader aware of a worsening world problem
(D) blame developing countries for deforestation
8. Which of the following best describes the tone of the passage?
(A) Concerned
(B) Disinterested
(C) Placid
(D) Exaggerated
9. This passage would probably be assigned reading in which of the following
courses?
(A) Geology
(B) Geography
(C) Geometry
(D) Marine Biology

REVIEW EXERCISE (SKILLS 1-6): Study each of the


passages and choose the best answers to the questions that
follow.

PASSAGE ONE (Questions 1-5)

Another noteworthy trend in twentieth-century music in the U.S. has


been the use of folk and popular music as a base for more serious
compositions. The motivation for these borrowings from traditional sources
might be a desire on the part of a composer to return to simpler forms, to
enhance patriotic feelings, or to establish an immediate rapport with an
audience. For whatever reason, composers such as Charles Ives- and Aaron

149
TOEFL Preparation Exercises Reading Comprehension & Vocabulary
Copland offered compositions featuring novel musical forms flavored with
refrains from traditional Americana. Ives employed the whole gamut of
patriotic songs, hymns, jazz, and popular songs in his compositions, while
Copland drew upon folk music, particularly as sources for his ballets Billy the
Kid, Rodeo, and Appalachian Spring.

1. The paragraph preceding this passage most probably discusses


(A) nineteenth-century music
(B) one development in music of this century
(C) the works of Aaron Copland
(D) the history of folk and popular music
2. Which of the following best describes the main idea of the passage?
(A) Traditional music has flavored some American musical compositions in this
century.
(B) Ives and Copland have used folk and popular music in their
compositions.
(C) A variety of explanations exist as to why a composer might use
traditional sources of music.
(D) Traditional music is composed of various types of folk and popular music.
3. It can be inferred from this passage that the author is not sure
(A) when Ives wrote his compositions
(B) that Ives and Copland actually borrowed from traditional music
(C) why certain composers borrowed from folk and popular music
(D) if Copland really featured new musical forms
4. Which of the following is not listed in the passage as a source for Ives'
compositions?
(A) National music
(B) Religious music
(C) Jazz
(D) American novels
5. The passage would most probably be assigned reading in which of the
following courses?
(A) American History
(B) The History of Jazz
(C) Modern American Music
(D) Composition

150
TOEFL Preparation Exercises Reading Comprehension & Vocabulary

PASSAGE TWO (Questions 6-10)


The rattlesnake has a reputation as a dangerous and deadly snake with a fierce
hatred for mankind. Although the rattlesnake is indeed a venomous snake capable
of killing a human, its nature has perhaps been somewhat-exaggerated in myth and
folklore.
The rattlesnake is not inherently aggressive and generally strikes only when it
has been put on the defensive. In its defensive posture the rattlesnake raises the
front part of its body off the ground and assumes an S-shaped form in preparation
for a lunge forward. At the end of a forward thrust, the rattlesnake pushes its fangs
into the victim, thereby injecting its venom.
There are more than thirty species of rattlesnakes, varying in length from 20
inches to six feet and also varying in toxicity of venom. In the United States there
are only a few deaths annually from rattlesnakes, with a mortality rate of less than 2
percent of those attacked.
6. Which of the following would be the best title for this passage?
(A) The Exaggerated Reputation of the Rattlesnake
(B) The Dangerous and Deadly Rattlesnake
(C) The Venomous Killer of Humans
(D) Myth and Folklore about Killers
7. According to the passage, which of the following is true about rattlesnakes?
(A) They are always ready to attack.
(B) They are always dangerous and deadly.
(C) Their fierce nature has been underplayed in myth and folklore.
(D) Their poison can kill people.
8. When a rattlesnake is ready to defend itself, it
(A) lies in an S-shape on the ground
(B) lunges with the back part of its body
(C) is partially off the ground
(D) assumes it is prepared by thrusting its fangs into the ground
9. It can be inferred from the passage that
(A) all rattlesnake bites are fatal
(B) all rattlesnake bites are not equally harmful
(C) the few deaths from rattlesnake bites are from the six-foot snakes
(D) deaths from rattlesnake bites have been steadily increasing
10. The author's purpose in this passage is to
(A) warn readers about the extreme danger from rattlesnakes
(B) explain a misconception about rattlesnakes
(C) describe a rattlesnake attack
(D) clarify how a rattlesnake kills humans
151
TOEFL Preparation Exercises Reading Comprehension & Vocabulary
PASSAGE THREE (Questions 11-15)
A massive banking crisis occurred in the United States in 1933. In the two
preceding years, a large number of banks had failed, and fear of lost savings
prompted many depositors to remove their funds from banks. Problems became so
serious in the state of Michigan that Governor William A. Comstock was forced to
declare a moratorium on all banking activities in the state on February 14, 1933.
The panic in Michigan quickly spread to other states, and by March 6, President
Franklin D. Roosevelt had declared a banking moratorium throughout the United
States that left the entire country without banking services.
Congress immediately met in a special session to solve the banking crisis and
on March 9 passed the Emergency Banking Act of 1933 to assist financially healthy
banks to reopen. By March 15, banks controlling 90 percent of the country's
financial reserves were again open for business.

11. The author's purpose in this passage is to


(A) discuss a problem and its resolution
(B) warn depositors about potential banking problems
(C) assess blame for a problem that had occurred
(D) praise Congress for its actions
12. The passage states that all the following occurred prior to 1933 EXCEPT that
(A) many banks went under
(B) many bank patrons were afraid of losing their deposits
(C) a lot of money was withdrawn from accounts
(D) Governor Comstock cancelled all banking activities in Michigan
13. The word "moratorium" (line 5) is closest in meaning to which of the following?
(A) Death
(B) Temporary cessation
(C) Murder
(D) Slow decline
14. Which of the following is implied in the passage?
(A) Congress did not give any special priority to the banking situation.
(B) The Emergency Banking Act helped all banks to reopen.
(C) Ninety percent of the banks reopened by the middle of March.
(D) Ten percent of the country's money was in financially unhealthy banks.

152
TOEFL Preparation Exercises Reading Comprehension & Vocabulary

15. Which of the following best describes the tone of the passage?
(A) Panicked
(B) Critical
(C) Historical
(D) Angry

153

You might also like